PrepScholar

Choose Your Test

Sat / act prep online guides and tips, how to write a perfect synthesis essay for the ap language exam.

author image

Advanced Placement (AP)

body-pencil-sharpen-notebook-1

If you're planning to take the AP Language (or AP Lang) exam , you might already know that 55% of your overall exam score will be based on three essays. The first of the three essays you'll have to write on the AP Language exam is called the "synthesis essay." If you want to earn full points on this portion of the AP Lang Exam, you need to know what a synthesis essay is and what skills are assessed by the AP Lang synthesis essay.

In this article, we'll explain the different aspects of the AP Lang synthesis essay, including what skills you need to demonstrate in your synthesis essay response in order to achieve a good score. We'll also give you a full breakdown of a real AP Lang Synthesis Essay prompt, provide an analysis of an AP Lang synthesis essay example, and give you four tips for how to write a synthesis essay.

Let's get started by taking a closer look at how the AP Lang synthesis essay works!

Synthesis Essay AP Lang: What It Is and How It Works

The AP Lang synthesis essay is the first of three essays included in the Free Response section of the AP Lang exam.

The AP Lang synthesis essay portion of the Free Response section lasts for one hour total . This hour consists of a recommended 15 minute reading period and a 40 minute writing period. Keep in mind that these time allotments are merely recommendations, and that exam takers can parse out the allotted 60 minutes to complete the synthesis essay however they choose.

Now, here's what the structure of the AP Lang synthesis essay looks like. The exam presents six to seven sources that are organized around a specific topic (like alternative energy or eminent domain, which are both past synthesis exam topics).

Of these six to seven sources, at least two are visual , including at least one quantitative source (like a graph or pie chart, for example). The remaining four to five sources are print text-based, and each one contains approximately 500 words.

In addition to six to seven sources, the AP Lang exam provides a written prompt that consists of three paragraphs. The prompt will briefly explain the essay topic, then present a claim that students will respond to in an essay that synthesizes material from at least three of the sources provided.

Here's an example prompt provided by the College Board:

Directions : The following prompt is based on the accompanying six sources.

This question requires you to integrate a variety of sources into a coherent, well-written essay. Refer to the sources to support your position; avoid mere paraphrase or summary. Your argument should be central; the sources should support this argument .

Remember to attribute both direct and indirect citations.

Introduction

Television has been influential in United States presidential elections since the 1960's. But just what is this influence, and how has it affected who is elected? Has it made elections fairer and more accessible, or has it moved candidates from pursuing issues to pursuing image?

Read the following sources (including any introductory information) carefully. Then, in an essay that synthesizes at least three of the sources for support, take a position that defends, challenges, or qualifies the claim that television has had a positive impact on presidential elections.

Refer to the sources as Source A, Source B, etc.; titles are included for your convenience.

Source A (Campbell) Source B (Hart and Triece) Source C (Menand) Source D (Chart) Source E (Ranney) Source F (Koppel)

Like we mentioned earlier, this prompt gives you a topic — which it briefly explains — then asks you to take a position. In this case, you'll have to choose a stance on whether television has positively or negatively affected U.S. elections. You're also given six sources to evaluate and use in your response. Now that you have everything you need, now your job is to write an amazing synthesis essay.

But what does "synthesize" mean, exactly? According to the CollegeBoard, when an essay prompt asks you to synthesize, it means that you should "combine different perspectives from sources to form a support of a coherent position" in writing. In other words, a synthesis essay asks you to state your claim on a topic, then highlight the relationships between several sources that support your claim on that topic. Additionally, you'll need to cite specific evidence from your sources to prove your point.

The synthesis essay counts for six of the total points on the AP Lang exam . Students can receive 0-1 points for writing a thesis statement in the essay, 0-4 based on incorporation of evidence and commentary, and 0-1 points based on sophistication of thought and demonstrated complex understanding of the topic.

You'll be evaluated based on how effectively you do the following in your AP Lang synthesis essay:

Write a thesis that responds to the exam prompt with a defensible position

Provide specific evidence that to support all claims in your line of reasoning from at least three of the sources provided, and clearly and consistently explain how the evidence you include supports your line of reasoning

Demonstrate sophistication of thought by either crafting a thoughtful argument, situating the argument in a broader context, explaining the limitations of an argument

Make rhetorical choices that strengthen your argument and/or employ a vivid and persuasive style throughout your essay.

If your synthesis essay meets the criteria above, then there's a good chance you'll score well on this portion of the AP Lang exam!

If you're looking for even more information on scoring, the College Board has posted the AP Lang Free Response grading rubric on its website. ( You can find it here. ) We recommend taking a close look at it since it includes additional details about the synthesis essay scoring.

body-chisel-break-apart

Don't be intimidated...we're going to teach you how to break down even the hardest AP synthesis essay prompt.

Full Breakdown of a Real AP Lang Synthesis Essay Prompt

In this section, we'll teach you how to analyze and respond to a synthesis essay prompt in five easy steps, including suggested time frames for each step of the process.

Step 1: Analyze the Prompt

The very first thing to do when the clock starts running is read and analyze the prompt. To demonstrate how to do this, we'll look at the sample AP Lang synthesis essay prompt below. This prompt comes straight from the 2018 AP Lang exam:

Eminent domain is the power governments have to acquire property from private owners for public use. The rationale behind eminent domain is that governments have greater legal authority over lands within their dominion than do private owners. Eminent domain has been instituted in one way or another throughout the world for hundreds of years.

Carefully read the following six sources, including the introductory information for each source. Then synthesize material from at least three of the sources and incorporate it into a coherent, well-developed essay that defends, challenges, or qualifies the notion that eminent domain is productive and beneficial.

Your argument should be the focus of your essay. Use the sources to develop your argument and explain the reasoning for it. Avoid merely summarizing the sources. Indicate clearly which sources you are drawing from, whether through direct quotation, paraphrase, or summary. You may cite the sources as Source A, Source B, etc., or by using the descriptions in parentheses.

On first read, you might be nervous about how to answer this prompt...especially if you don't know what eminent domain is! But if you break the prompt down into chunks, you'll be able to figure out what the prompt is asking you to do in no time flat.

To get a full understanding of what this prompt wants you to do, you need to identify the most important details in this prompt, paragraph by paragraph. Here's what each paragraph is asking you to do:

  • Paragraph 1: The prompt presents and briefly explains the topic that you'll be writing your synthesis essay about. That topic is the concept of eminent domain.
  • Paragraph 2: The prompt presents a specific claim about the concept of eminent domain in this paragraph: Eminent domain is productive and beneficial. This paragraph instructs you to decide whether you want to defend, challenge, or qualify that claim in your synthesis essay , and use material from at least three of the sources provided in order to do so.
  • Paragraph 3: In the last paragraph of the prompt, the exam gives you clear instructions about how to approach writing your synthesis essay . First, make your argument the focus of the essay. Second, use material from at least three of the sources to develop and explain your argument. Third, provide commentary on the material you include, and provide proper citations when you incorporate quotations, paraphrases, or summaries from the sources provided.

So basically, you'll have to agree with, disagree with, or qualify the claim stated in the prompt, then use at least three sources substantiate your answer. Since you probably don't know much about eminent domain, you'll probably decide on your position after you read the provided sources.

To make good use of your time on the exam, you should spend around 2 minutes reading the prompt and making note of what it's asking you to do. That will leave you plenty of time to read the sources provided, which is the next step to writing a synthesis essay.

Step 2: Read the Sources Carefully

After you closely read the prompt and make note of the most important details, you need to read all of the sources provided. It's tempting to skip one or two sources to save time--but we recommend you don't do this. That's because you'll need a thorough understanding of the topic before you can accurately address the prompt!

For the sample exam prompt included above, there are six sources provided. We're not going to include all of the sources in this article, but you can view the six sources from this question on the 2018 AP Lang exam here . The sources include five print-text sources and one visual source, which is a cartoon.

As you read the sources, it's important to read quickly and carefully. Don't rush! Keep your pencil in hand to quickly mark important passages that you might want to use as evidence in your synthesis. While you're reading the sources and marking passages, you want to think about how the information you're reading influences your stance on the issue (in this case, eminent domain).

When you finish reading, take a few seconds to summarize, in a phrase or sentence, whether the source defends, challenges, or qualifies whether eminent domain is beneficial (which is the claim in the prompt) . Though it might not feel like you have time for this, it's important to give yourself these notes about each source so you know how you can use each one as evidence in your essay.

Here's what we mean: say you want to challenge the idea that eminent domain is useful. If you've jotted down notes about each source and what it's saying, it will be easier for you to pull the relevant information into your outline and your essay.

So how much time should you spend reading the provided sources? The AP Lang exam recommends taking 15 minutes to read the sources . If you spend around two of those minutes reading and breaking down the essay prompt, it makes sense to spend the remaining 13 minutes reading and annotating the sources.

If you finish reading and annotating early, you can always move on to drafting your synthesis essay. But make sure you're taking your time and reading carefully! It's better to use a little extra time reading and understanding the sources now so that you don't have to go back and re-read the sources later.

body-weightlifting-lift-strong

A strong thesis will do a lot of heavy lifting in your essay. (See what we did there?)

Step 3: Write a Strong Thesis Statement

After you've analyzed the prompt and thoroughly read the sources, the next thing you need to do in order to write a good synthesis essay is write a strong thesis statement .

The great news about writing a thesis statement for this synthesis essay is that you have all the tools you need to do it at your fingertips. All you have to do in order to write your thesis statement is decide what your stance is in relationship to the topic provided.

In the example prompt provided earlier, you're essentially given three choices for how to frame your thesis statement: you can either defend, challenge, or qualify a claim that's been provided by the prompt, that eminent domain is productive and beneficial . Here's what that means for each option:

If you choose to defend the claim, your job will be to prove that the claim is correct . In this case, you'll have to show that eminent domain is a good thing.

If you choose to challenge the claim, you'll argue that the claim is incorrect. In other words, you'll argue that eminent domain isn't productive or beneficial.

If you choose to qualify, that means you'll agree with part of the claim, but disagree with another part of the claim. For instance, you may argue that eminent domain can be a productive tool for governments, but it's not beneficial for property owners. Or maybe you argue that eminent domain is useful in certain circumstances, but not in others.

When you decide whether you want your synthesis essay to defend, challenge, or qualify that claim, you need to convey that stance clearly in your thesis statement. You want to avoid simply restating the claim provided in the prompt, summarizing the issue without making a coherent claim, or writing a thesis that doesn't respond to the prompt.

Here's an example of a thesis statement that received full points on the eminent domain synthesis essay:

Although eminent domain can be misused to benefit private interests at the expense of citizens, it is a vital tool of any government that intends to have any influence on the land it governs beyond that of written law.

This thesis statement received full points because it states a defensible position and establishes a line of reasoning on the issue of eminent domain. It states the author's position (that some parts of eminent domain are good, but others are bad), then goes on to explain why the author thinks that (it's good because it allows the government to do its job, but it's bad because the government can misuse its power.)

Because this example thesis statement states a defensible position and establishes a line of reasoning, it can be elaborated upon in the body of the essay through sub-claims, supporting evidence, and commentary. And a solid argument is key to getting a six on your synthesis essay for AP Lang!

Looking for help studying for your AP exam?

Our one-on-one online AP tutoring services can help you prepare for your AP exams. Get matched with a top tutor who got a high score on the exam you're studying for!

Get a 5 On Your AP Exam

Step 4: Create a Bare-Bones Essay Outline

Once you've got your thesis statement drafted, you have the foundation you need to develop a bare bones outline for your synthesis essay. Developing an outline might seem like it's a waste of your precious time, but if you develop your outline well, it will actually save you time when you start writing your essay.

With that in mind, we recommend spending 5 to 10 minutes outlining your synthesis essay . If you use a bare-bones outline like the one below, labeling each piece of content that you need to include in your essay draft, you should be able to develop out the most important pieces of the synthesis before you even draft the actual essay.

To help you see how this can work on test day, we've created a sample outline for you. You can even memorize this outline to help you out on test day! In the outline below, you'll find places to fill in a thesis statement, body paragraph topic sentences, evidence from the sources provided, and commentary :

  • Present the context surrounding the essay topic in a couple of sentences (this is a good place to use what you learned about the major opinions or controversies about the topic from reading your sources).
  • Write a straightforward, clear, and concise thesis statement that presents your stance on the topic
  • Topic sentence presenting first supporting point or claim
  • Evidence #1
  • Commentary on Evidence #1
  • Evidence #2 (if needed)
  • Commentary on Evidence #2 (if needed)
  • Topic sentence presenting second supporting point or claim
  • Topic sentence presenting three supporting point or claim
  • Sums up the main line of reasoning that you developed and defended throughout the essay
  • Reiterates the thesis statement

Taking the time to develop these crucial pieces of the synthesis in a bare-bones outline will give you a map for your final essay. Once you have a map, writing the essay will be much easier.

Step 5: Draft Your Essay Response

The great thing about taking a few minutes to develop an outline is that you can develop it out into your essay draft. After you take about 5 to 10 minutes to outline your synthesis essay, you can use the remaining 30 to 35 minutes to draft your essay and review it.

Since you'll outline your essay before you start drafting, writing the essay should be pretty straightforward. You'll already know how many paragraphs you're going to write, what the topic of each paragraph will be, and what quotations, paraphrases, or summaries you're going to include in each paragraph from the sources provided. You'll just have to fill in one of the most important parts of your synthesis—your commentary.

Commentaries are your explanation of why your evidence supports the argument you've outlined in your thesis. Your commentary is where you actually make your argument, which is why it's such a critical part of your synthesis essay.

When thinking about what to say in your commentary, remember one thing the AP Lang synthesis essay prompt specifies: don't just summarize the sources. Instead, as you provide commentary on the evidence you incorporate, you need to explain how that evidence supports or undermines your thesis statement . You should include commentary that offers a thoughtful or novel perspective on the evidence from your sources to develop your argument.

One very important thing to remember as you draft out your essay is to cite your sources. The AP Lang exam synthesis essay prompt indicates that you can use generic labels for the sources provided (e.g. "Source 1," "Source 2," "Source 3," etc.). The exam prompt will indicate which label corresponds with which source, so you'll need to make sure you pay attention and cite sources accurately. You can cite your sources in the sentence where you introduce a quote, summary, or paraphrase, or you can use a parenthetical citation. Citing your sources affects your score on the synthesis essay, so remembering to do this is important.

body-green-arrow-down

Keep reading for a real-life example of a great AP synthesis essay response!

Real-Life AP Synthesis Essay Example and Analysis

If you're still wondering how to write a synthesis essay, examples of real essays from past AP Lang exams can make things clearer. These real-life student AP synthesis essay responses can be great for helping you understand how to write a synthesis essay that will knock the graders' socks off .

While there are multiple essay examples online, we've chosen one to take a closer look at. We're going to give you a brief analysis of one of these example student synthesis essays from the 2019 AP Lang Exam below!

Example Synthesis Essay AP Lang Response

To get started, let's look at the official prompt for the 2019 synthesis essay:

In response to our society's increasing demand for energy, large-scale wind power has drawn attention from governments and consumers as a potential alternative to traditional materials that fuel our power grids, such as coal, oil, natural gas, water, or even newer sources such as nuclear or solar power. Yet the establishment of large-scale, commercial-grade wind farms is often the subject of controversy for a variety of reasons.

Carefully read the six sources, found on the AP English Language and Composition 2019 Exam (Question 1), including the introductory information for each source. Write an essay that synthesizes material from at least three of the sources and develops your position on the most important factors that an individual or agency should consider when deciding whether to establish a wind farm.

Source A (photo) Source B (Layton) Source C (Seltenrich) Source D (Brown) Source E (Rule) Source F (Molla)

In your response you should do the following:

  • Respond to the prompt with a thesis presents a defensible position.
  • Select and use evidence from at least 3 of the provided sources to support your line of reasoning. Indicate clearly the sources used through direct quotation, paraphrase, or summary. Sources may be cited as Source A, Source B, etc., or by using the description in parentheses.
  • Explain how the evidence supports your line of reasoning.
  • Use appropriate grammar and punctuation in communicating your argument.

Now that you know exactly what the prompt asked students to do on the 2019 AP Lang synthesis essay, here's an AP Lang synthesis essay example, written by a real student on the AP Lang exam in 2019:

[1] The situation has been known for years, and still very little is being done: alternative power is the only way to reliably power the changing world. The draw of power coming from industry and private life is overwhelming current sources of non-renewable power, and with dwindling supplies of fossil fuels, it is merely a matter of time before coal and gas fuel plants are no longer in operation. So one viable alternative is wind power. But as with all things, there are pros and cons. The main factors for power companies to consider when building wind farms are environmental boon, aesthetic, and economic factors.

[2] The environmental benefits of using wind power are well-known and proven. Wind power is, as qualified by Source B, undeniably clean and renewable. From their production requiring very little in the way of dangerous materials to their lack of fuel, besides that which occurs naturally, wind power is by far one of the least environmentally impactful sources of power available. In addition, wind power by way of gearbox and advanced blade materials, has the highest percentage of energy retention. According to Source F, wind power retains 1,164% of the energy put into the system – meaning that it increases the energy converted from fuel (wind) to electricity 10 times! No other method of electricity production is even half that efficient. The efficiency and clean nature of wind power are important to consider, especially because they contribute back to power companies economically.

[3] Economically, wind power is both a boon and a bone to electric companies and other users. For consumers, wind power is very cheap, leading to lower bills than from any other source. Consumers also get an indirect reimbursement by way of taxes (Source D). In one Texan town, McCamey, tax revenue increased 30% from a wind farm being erected in the town. This helps to finance improvements to the town. But, there is no doubt that wind power is also hurting the power companies. Although, as renewable power goes, wind is incredibly cheap, it is still significantly more expensive than fossil fuels. So, while it is helping to cut down on emissions, it costs electric companies more than traditional fossil fuel plants. While the general economic trend is positive, there are some setbacks which must be overcome before wind power can take over as truly more effective than fossil fuels.

[4] Aesthetics may be the greatest setback for power companies. Although there may be significant economic and environmental benefit to wind power, people will always fight to preserve pure, unspoiled land. Unfortunately, not much can be done to improve the visual aesthetics of the turbines. White paint is the most common choice because it "[is] associated with cleanliness." (Source E). But, this can make it stand out like a sore thumb, and make the gargantuan machines seem more out of place. The site can also not be altered because it affects generating capacity. Sound is almost worse of a concern because it interrupts personal productivity by interrupting people's sleep patterns. One thing for power companies to consider is working with turbine manufacturing to make the machines less aesthetically impactful, so as to garner greater public support.

[5] As with most things, wind power has no easy answer. It is the responsibility of the companies building them to weigh the benefits and the consequences. But, by balancing economics, efficiency, and aesthetics, power companies can create a solution which balances human impact with environmental preservation.

And that's an entire AP Lang synthesis essay example, written in response to a real AP Lang exam prompt! It's important to remember AP Lang exam synthesis essay prompts are always similarly structured and worded, and students often respond in around the same number of paragraphs as what you see in the example essay response above.

Next, let's analyze this example essay and talk about what it does effectively, where it could be improved upon, and what score past exam scorers awarded it.

To get started on an analysis of the sample synthesis essay, let's look at the scoring commentary provided by the College Board:

  • For development of thesis, the essay received 1 out of 1 possible points
  • For evidence and commentary, the essay received 4 out of 4 possible points
  • For sophistication of thought, the essay received 0 out of 1 possible points.

This means that the final score for this example essay was a 5 out of 6 possible points . Let's look more closely at the content of the example essay to figure out why it received this score breakdown.

Thesis Development

The thesis statement is one of the three main categories that is taken into consideration when you're awarded points on this portion of the exam. This sample essay received 1 out of 1 total points.

Now, here's why: the thesis statement clearly and concisely conveys a position on the topic presented in the prompt--alternative energy and wind power--and defines the most important factors that power companies should consider when deciding whether to establish a wind farm.

Evidence and Commentary

The second key category taken into consideration when synthesis exams are evaluated is incorporation of evidence and commentary. This sample received 4 out of 4 possible points for this portion of the synthesis essay. At bare minimum, this sample essay meets the requirement mentioned in the prompt that the writer incorporate evidence from at least three of the sources provided.

On top of that, the writer does a good job of connecting the incorporated evidence back to the claim made in the thesis statement through effective commentary. The commentary in this sample essay is effective because it goes beyond just summarizing what the provided sources say. Instead, it explains and analyzes the evidence presented in the selected sources and connects them back to supporting points the writer makes in each body paragraph.

Finally, the writer of the essay also received points for evidence and commentary because the writer developed and supported a consistent line of reasoning throughout the essay . This line of reasoning is summed up in the fourth paragraph in the following sentence: "One thing for power companies to consider is working with turbine manufacturing to make the machines less aesthetically impactful, so as to garner greater public support."

Because the writer did a good job consistently developing their argument and incorporating evidence, they received full marks in this category. So far, so good!

Sophistication of Thought

Now, we know that this essay received a score of 5 out of 6 total points, and the place where the writer lost a point was on the basis of sophistication of thought, for which the writer received 0 out of 1 points. That's because this sample essay makes several generalizations and vague claims where it could have instead made specific claims that support a more balanced argument.

For example, in the following sentence from the 5th paragraph of the sample essay, the writer misses the opportunity to state specific possibilities that power companies should consider for wind energy . Instead, the writer is ambiguous and non-committal, saying, "As with most things, wind power has no easy answer. It is the responsibility of the companies building them to weigh the benefits and consequences."

If the writer of this essay was interested in trying to get that 6th point on the synthesis essay response, they could consider making more specific claims. For instance, they could state the specific benefits and consequences power companies should consider when deciding whether to establish a wind farm. These could include things like environmental impacts, economic impacts, or even population density!

Despite losing one point in the last category, this example synthesis essay is a strong one. It's well-developed, thoughtfully written, and advances an argument on the exam topic using evidence and support throughout.

body-number-four-post-it-note

4 Tips for How to Write a Synthesis Essay

AP Lang is a timed exam, so you have to pick and choose what you want to focus on in the limited time you're given to write the synthesis essay. Keep reading to get our expert advice on what you should focus on during your exam.

Tip 1: Read the Prompt First

It may sound obvious, but when you're pressed for time, it's easy to get flustered. Just remember: when it comes time to write the synthesis essay, read the prompt first !

Why is it so important to read the prompt before you read the sources? Because when you're aware of what kind of question you're trying to answer, you'll be able to read the sources more strategically. The prompt will help give you a sense of what claims, points, facts, or opinions to be looking for as you read the sources.

Reading the sources without having read the prompt first is kind of like trying to drive while wearing a blindfold: you can probably do it, but it's likely not going to end well!

Tip 2: Make Notes While You Read

During the 15-minute reading period at the beginning of the synthesis essay, you'll be reading through the sources as quickly as you can. After all, you're probably anxious to start writing!

While it's definitely important to make good use of your time, it's also important to read closely enough that you understand your sources. Careful reading will allow you to identify parts of the sources that will help you support your thesis statement in your essay, too.

As you read the sources, consider marking helpful passages with a star or check mark in the margins of the exam so you know which parts of the text to quickly re-read as you form your synthesis essay. You might also consider summing up the key points or position of each source in a sentence or a few words when you finish reading each source during the reading period. Doing so will help you know where each source stands on the topic given and help you pick the three (or more!) that will bolster your synthesis argument.

Tip 3: Start With the Thesis Statement

If you don't start your synthesis essay with a strong thesis statement, it's going to be tough to write an effective synthesis essay. As soon as you finish reading and annotating the provided sources, the thing you want to do next is write a strong thesis statement.

According to the CollegeBoard grading guidelines for the AP Lang synthesis essay, a strong thesis statement will respond to the prompt— not restate or rephrase the prompt. A good thesis will take a clear, defensible position on the topic presented in the prompt and the sources.

In other words, to write a solid thesis statement to guide the rest of your synthesis essay, you need to think about your position on the topic at hand and then make a claim about the topic based on your position. This position will either be defending, challenging, or qualifying the claim made in the essay's prompt.

The defensible position that you establish in your thesis statement will guide your argument in the rest of the essay, so it's important to do this first. Once you have a strong thesis statement, you can begin outlining your essay.

Tip 4: Focus on Your Commentary

Writing thoughtful, original commentary that explains your argument and your sources is important. In fact, doing this well will earn you four points (out of a total of six)!

AP Lang provides six to seven sources for you on the exam, and you'll be expected to incorporate quotations, paraphrases, or summaries from at least three of those sources into your synthesis essay and interpret that evidence for the reader.

While incorporating evidence is very important, in order to get the extra point for "sophistication of thought" on the synthesis essay, it's important to spend more time thinking about your commentary on the evidence you choose to incorporate. The commentary is your chance to show original thinking, strong rhetorical skills, and clearly explain how the evidence you've included supports the stance you laid out in your thesis statement.

To earn the 6th possible point on the synthesis essay, make sure your commentary demonstrates a nuanced understanding of the source material, explains this nuanced understanding, and places the evidence incorporated from the sources in conversation with each other. To do this, make sure you're avoiding vague language. Be specific when you can, and always tie your commentary back to your thesis!

body-person-arrows-next

What's Next?

There's a lot more to the AP Language exam than just the synthesis essay. Be sure to check out our expert guide to the entire exam , then learn more about the tricky multiple choice section .

Is the AP Lang exam hard...or is it easy? See how it stacks up to other AP tests on our list of the hardest AP exams .

Did you know there are technically two English AP exams? You can learn more about the second English AP test, the AP Literature exam, in this article . And if you're confused about whether you should take the AP Lang or AP Lit test , we can help you make that decision, too.

Want to improve your SAT score by 160 points or your ACT score by 4 points? We've written a guide for each test about the top 5 strategies you must be using to have a shot at improving your score. Download it for free now:

Get eBook: 5 Tips for 160+ Points

Ashley Sufflé Robinson has a Ph.D. in 19th Century English Literature. As a content writer for PrepScholar, Ashley is passionate about giving college-bound students the in-depth information they need to get into the school of their dreams.

Student and Parent Forum

Our new student and parent forum, at ExpertHub.PrepScholar.com , allow you to interact with your peers and the PrepScholar staff. See how other students and parents are navigating high school, college, and the college admissions process. Ask questions; get answers.

Join the Conversation

Ask a Question Below

Have any questions about this article or other topics? Ask below and we'll reply!

Improve With Our Famous Guides

  • For All Students

The 5 Strategies You Must Be Using to Improve 160+ SAT Points

How to Get a Perfect 1600, by a Perfect Scorer

Series: How to Get 800 on Each SAT Section:

Score 800 on SAT Math

Score 800 on SAT Reading

Score 800 on SAT Writing

Series: How to Get to 600 on Each SAT Section:

Score 600 on SAT Math

Score 600 on SAT Reading

Score 600 on SAT Writing

Free Complete Official SAT Practice Tests

What SAT Target Score Should You Be Aiming For?

15 Strategies to Improve Your SAT Essay

The 5 Strategies You Must Be Using to Improve 4+ ACT Points

How to Get a Perfect 36 ACT, by a Perfect Scorer

Series: How to Get 36 on Each ACT Section:

36 on ACT English

36 on ACT Math

36 on ACT Reading

36 on ACT Science

Series: How to Get to 24 on Each ACT Section:

24 on ACT English

24 on ACT Math

24 on ACT Reading

24 on ACT Science

What ACT target score should you be aiming for?

ACT Vocabulary You Must Know

ACT Writing: 15 Tips to Raise Your Essay Score

How to Get Into Harvard and the Ivy League

How to Get a Perfect 4.0 GPA

How to Write an Amazing College Essay

What Exactly Are Colleges Looking For?

Is the ACT easier than the SAT? A Comprehensive Guide

Should you retake your SAT or ACT?

When should you take the SAT or ACT?

Stay Informed

synthesis ap lang format

Get the latest articles and test prep tips!

Looking for Graduate School Test Prep?

Check out our top-rated graduate blogs here:

GRE Online Prep Blog

GMAT Online Prep Blog

TOEFL Online Prep Blog

Holly R. "I am absolutely overjoyed and cannot thank you enough for helping me!”

What are your chances of acceptance?

Calculate for all schools, your chance of acceptance.

Duke University

Your chancing factors

Extracurriculars.

synthesis ap lang format

How to Write the AP Lang Synthesis Essay + Example

Do you know how to improve your profile for college applications.

See how your profile ranks among thousands of other students using CollegeVine. Calculate your chances at your dream schools and learn what areas you need to improve right now — it only takes 3 minutes and it's 100% free.

Show me what areas I need to improve

What’s Covered:

What is the ap lang synthesis essay, how will ap scores affect my college chances.

AP English Language and Composition, commonly known as AP Lang, is one of the most engaging and popular AP classes offered at most high schools, with over 535,000 students taking the class . AP Lang tests your ability to analyze written pieces, synthesize information, write rhetorical essays, and create cohesive and concrete arguments. However, the class is rather challenging as only 62% of students were able to score a three or higher on the exam. 

The AP Lang exam has two sections. The first consists of 45 multiple choice questions which need to be completed in an hour. This portion counts for around 45% of your total score. These questions ask students to analyze written pieces and answer questions related to each respective passage.  All possible answer choices can be found within the text, and no prior knowledge of literature is needed to understand the passages.

The second section contains three free-response questions to be finished in under two hours and 15 minutes. This section counts for 55% of your score and includes the synthesis essay, the rhetorical essay, and the argumentative essay.

  • The synthesis essay requires you to read 6-7 sources and create an argument using at least three sources.
  • The rhetorical analysis essay requires you to describe how a piece of writing evokes specific meanings and symbolism.
  • The argumentative essay requires you to pick a perspective of a debate and create an argument based on the evidence provided.

In this post, we will take a look at the AP Lang synthesis essay and discuss tips and tricks to master this part of the exam. We will also provide an example of a well-written essay for review.  

The AP Lang synthesis essay is the first of three essays included in the Free Response section of the AP Lang exam. The exam presents 6-7 sources that are organized around a specific topic, with two of those sources purely visual, including a single quantitative source (like a graph or pie chart). The remaining 4-5 sources are text-based, containing around 500 words each. It’s recommended that students spend an hour on this essay—15 minute reading period, 40 minutes writing, and 5 minutes of spare time to check over work.

Each synthesis essay has a topic that all the sources will relate to. A prompt will explaining the topic and provide some background, although the topics are usually broad so you will probably know something related to the issue. It will also present a claim that students will respond to in an essay format using information from at least three of the provided sources. You will need to take a stance, either agreeing or disagreeing with the position provided in the claim. 

According to the CollegeBoard, they are looking for essays that “combine different perspectives from sources to form a support of a coherent position.” This means that you must state your claim on the topic and highlight relationships between several sources that support your specific position on the topic. Additionally, you’ll need to cite clear evidence from your sources to prove your point.

The synthesis essay counts for six points on the AP Lang exam. Students can receive 0-1 points for writing a thesis statement, 0-4 based on the incorporation of evidence and commentary, and 0-1 points based on the sophistication of thought and demonstration of complex understanding.

While this essay seems extremely overwhelming, considering there are a total of three free-response essays to complete, with proper time management and practiced skills, this essay is manageable and straightforward. In order to enhance the time management aspect of the test to the best of your ability, it is essential to divide the essay up into five key steps.

Step 1: Analyze the Prompt

As soon as the clock starts, carefully read and analyze what the prompt asks from you. It might be helpful to markup the text to identify the most critical details. You should only spend around 2 minutes reading the prompt so you have enough time to read all the sources and figure out your argument. Don’t feel like you need to immediately pick your stance on the claim right after reading the prompt. You should read the sources before you commit to your argument.

Step 2: Read the Sources Carefully

Although you are only required to use 3 of the 6-7 sources provides, make sure you read ALL of the sources. This will allow you to better understand the topic and make the most educated decision of which sources to use in your essay. Since there are a lot of sources to get through, you will need to read quickly and carefully.

Annotating will be your best friend during the reading period. Highlight and mark important concepts or lines from each passage that would be helpful in your essay. Your argument will probably begin forming in your head as you go through the passages, so you will save yourself a lot of time later on if you take a few seconds to write down notes in the margins. After you’ve finished reading a source, reflect on whether the source defends, challenges, or qualifies your argument.

You will have around 13 minutes to read through all the sources, but it’s very possible you will finish earlier if you are a fast reader. Take the leftover time to start developing your thesis and organizing your thoughts into an outline so you have more time to write. 

Step 3: Write a Strong Thesis Statement 

In order to write a good thesis statement, all you have to do is decide your stance on the claim provided in the prompt and give an overview of your evidence. You essentially have three choices on how to frame your thesis statement: You can defend, challenge or qualify a claim that’s been provided by the prompt. 

  • If you are defending the claim, your job will be to prove that the claim is correct .
  • If you are challenging the claim, your job will be to prove that the claim is incorrect .
  • If you choose to qualify the claim, your job will be to agree to a part of the claim and disagree with another part of the claim. 

A strong thesis statement will clearly state your stance without summarizing the issue or regurgitating the claim. The CollegeBoard is looking for a thesis statement that “states a defensible position and establishes a line of reasoning on the issue provided in the prompt.”

Step 4: Create a Minimal Essay Outline

Developing an outline might seem like a waste of time when you are up against the clock, but believe us, taking 5-10 minutes to outline your essay will be much more useful in the long run than jumping right into the essay.

Your outline should include your thesis statement and three main pieces of evidence that will constitute each body paragraph. Under each piece of evidence should be 2-3 details from the sources that you will use to back up your claim and some commentary on how that evidence proves your thesis.

Step 5: Write your Essay

Use the remaining 30-35 minutes to write your essay. This should be relatively easy if you took the time to mark up the sources and have a detailed outline.  Remember to add special consideration and emphasis to the commentary sections of the supporting arguments outlined in your thesis. These sentences are critical to the overall flow of the essay and where you will be explaining how the evidence supports or undermines the claim in the prompt.

Also, when referencing your sources, write the in-text citations as follows: “Source 1,” “Source 2,” “Source 3,” etc. Make sure to pay attention to which source is which in order to not incorrectly cite your sources. In-text citations will impact your score on the essay and are an integral part of the process.

After you finish writing, read through your essay for any grammatical errors or mistakes before you move onto the next essay.

Here are six must-have tips and tricks to get a good score on the synthesis essay:

  • Cite at least four sources , even though the minimum requirement is three. Remember not to plagiarize and cite everything you use in your arguments.
  • Make sure to develop a solid and clear thesis . Develop a stable stance for the claim and stick with it throughout the entire paper.
  • Don’t summarize the sources. The summary of the sources does not count as an argument. 
  • You don’t necessarily have to agree with the sources in order to cite them. Using a source to support a counterargument is still a good use of a source.
  • Cite the sources that you understand entirely . If you don’t, it could come back to bite you in the end. 
  • Use small quotes , do not quote entire paragraphs. Make sure the quote does not disrupt the flow or grammar of the sentence you write. 

synthesis ap lang format

Discover your chances at hundreds of schools

Our free chancing engine takes into account your history, background, test scores, and extracurricular activities to show you your real chances of admission—and how to improve them.

Here is an example prompt and essay from 2019 that received 5 of the 6 total points available:

In response to our society’s increasing demand for energy, large-scale wind power has drawn attention from governments and consumers as a potential alternative to traditional materials that fuel our power grids, such as coal, oil, natural gas, water, or even newer sources such as nuclear or solar power. Yet the establishment of large-scale, commercial-grade wind farms is often the subject of controversy for a variety of reasons.

Carefully read the six sources, found on the AP English Language and Composition 2019 Exam (Question 1), including the introductory information for each source. Write an essay that synthesizes material from at least three of the sources and develops your position on the most important factors that an individual or agency should consider when deciding whether to establish a wind farm.

Source A (photo)

Source B (Layton)

Source C (Seltenrich)

Source D (Brown)

Source E (Rule)

Source F (Molla)

In your response you should do the following:

  • Respond to the prompt with a thesis presents a defensible position.
  • Select and use evidence from at least 3 of the provided sources to support your line of reasoning. Indicate clearly the sources used through direct quotation, paraphrase, or summary. Sources may be cited as Source A, Source B, etc., or by using the description in parentheses.
  • Explain how the evidence supports your line of reasoning.
  • Use appropriate grammar and punctuation in communicating your argument.

[1] The situation has been known for years, and still very little is being done: alternative power is the only way to reliably power the changing world. The draw of power coming from industry and private life is overwhelming current sources of non-renewable power, and with dwindling supplies of fossil fuels, it is merely a matter of time before coal and gas fuel plants are no longer in operation. So one viable alternative is wind power. But as with all things, there are pros and cons. The main factors for power companies to consider when building wind farms are environmental boon, aesthetic, and economic factors.

[2] The environmental benefits of using wind power are well-known and proven. Wind power is, as qualified by Source B, undeniably clean and renewable. From their production requiring very little in the way of dangerous materials to their lack of fuel, besides that which occurs naturally, wind power is by far one of the least environmentally impactful sources of power available. In addition, wind power by way of gearbox and advanced blade materials, has the highest percentage of energy retention. According to Source F, wind power retains 1,164% of the energy put into the system – meaning that it increases the energy converted from fuel (wind) to electricity 10 times! No other method of electricity production is even half that efficient. The efficiency and clean nature of wind power are important to consider, especially because they contribute back to power companies economically.

[3] Economically, wind power is both a boon and a bone to electric companies and other users. For consumers, wind power is very cheap, leading to lower bills than from any other source. Consumers also get an indirect reimbursement by way of taxes (Source D). In one Texan town, McCamey, tax revenue increased 30% from a wind farm being erected in the town. This helps to finance improvements to the town. But, there is no doubt that wind power is also hurting the power companies. Although, as renewable power goes, wind is incredibly cheap, it is still significantly more expensive than fossil fuels. So, while it is helping to cut down on emissions, it costs electric companies more than traditional fossil fuel plants. While the general economic trend is positive, there are some setbacks which must be overcome before wind power can take over as truly more effective than fossil fuels.

[4] Aesthetics may be the greatest setback for power companies. Although there may be significant economic and environmental benefit to wind power, people will always fight to preserve pure, unspoiled land. Unfortunately, not much can be done to improve the visual aesthetics of the turbines. White paint is the most common choice because it “[is] associated with cleanliness.” (Source E). But, this can make it stand out like a sore thumb, and make the gargantuan machines seem more out of place. The site can also not be altered because it affects generating capacity. Sound is almost worse of a concern because it interrupts personal productivity by interrupting people’s sleep patterns. One thing for power companies to consider is working with turbine manufacturing to make the machines less aesthetically impactful, so as to garner greater public support.

[5] As with most things, wind power has no easy answer. It is the responsibility of the companies building them to weigh the benefits and the consequences. But, by balancing economics, efficiency, and aesthetics, power companies can create a solution which balances human impact with environmental preservation.

More examples can be found here at College Board.

While AP Scores help to boost your weighted GPA, or give you the option to get college credit, AP Scores don’t have a strong effect on your admissions chances . However, colleges can still see your self-reported scores, so you might not want to automatically send scores to colleges if they are lower than a 3. That being said, admissions officers care far more about your grade in an AP class than your score on the exam.

Related CollegeVine Blog Posts

synthesis ap lang format

How to Write the AP Lang Synthesis Essay with Example

September 5, 2023

AP Lang synthesis essay, AP Language

If you’re highly interested in learning more about writing analysis, then chances are you enrolled in AP Lang. Essentially, AP Lang is an advanced course for high schoolers that combines interest and knowledge in English with critical thinking. In the class, students learn how to analyze and synthesize a variety of texts to construct well-reasoned arguments. If you take AP Lang, then you can opt to take the AP test at the conclusion of the school year. On the exam, students write the AP Lang synthesis essay to demonstrate their learned abilities. In this article, we’ll look at what the AP Lang synthesis essay requires and show an example to provide better understanding of what to expect on the exam.

AP Lang Exam Basics

The AP Lang exam is separated into two sections. In the first section, students have one hour to answer a series of 45 multiple-choice questions. Here, about half of the questions are based on passages students read. The other half are focused on the best revision techniques. Essentially, the answers for the latter 20-22 questions are geared toward revising mock essays.

In this article, however, we’ll focus mainly on the second part of the exam: the AP Lang synthesis essay.

In this second section, students have two hours and 15 minutes to write three essays of their own design. The three open-ended questions in this section are intended to be free-response and allow for a variety of approaches. Each question is intended to allow up to 40 minutes to complete.

For the AP Lang synthesis essay, students are presented with a scenario of the College Board’s design. The scenario will provide its own thesis statement. Usually, scenarios relate to real-world problems like environmental concerns, media, or government policies.

For each scenario, students are provided with 6-7 outside sources. These sources could be in the form of an image, visual graph, or written paragraph. For written paragraphs, the sources are usually no more than 500 words.

Students are then expected to incorporate at least 3-4 of these outside sources into their essay response. The outside sources are intended to be used as supporting evidence for the student’s chosen stance or argument. Students are able to either agree with or disagree with the thesis presented in the original scenario.

AP Lang Exam – Scoring

In the second part of the AP Lang exam, students can earn a possible 6 points on each essay. 1 point is earned for the development of a thesis. Up to 4 points can be earned for evidence and commentary. The final 1 point is earned for sophistication of thought.

AP Lang Exam – Takeaways

Ultimately, the goal of the AP Lang synthesis essay is not whether the student is “right” or “wrong” in their argument. The key is that students are able to reasonably and clearly support their argument using the provided sources as evidence .

The College Board looks for your ability to identify relationships between texts , form a coherent argument , and interpret external sources .

Synthesis Essay AP Lang Examples

If you’re not sure how the questions will look on the AP Lang synthesis essay section, we’ll provide an example. After the example, we’ll break down the strengths and weaknesses of the response. That way, you’ll have a better idea of what the College Board is looking for.

Additionally, the College Board has released previous AP Lang synthesis essay examples you can review. They even have essay questions as recent as 2022 . For further support, a scoring commentary and comments from the Chief Reader are also available to view. Additionally, there are other examples you can view from earlier years .

Note: A good strategy to study for the synthesis essay AP Lang exam is to review your rhetorical devices and literary devices . Understanding how these devices function can be essential in constructing a cohesive essay.

Synthesis Essay AP Lang Examples – Sample Question

Below is a sample question from the AP Lang synthesis essay and a response to the prompt. This question was taken directly from a 2022 exam . However, the response to the question will be originally crafted for the purpose of this newsletter. As well, all supporting evidence will be originally created and does not correspond to any previous test.

The Question

Since the early 2000s, the United States government and a number of corporations have sponsored initiatives to improve education in the STEM disciplines: science, technology, engineering, and mathematics. The emphasis on STEM subjects in elementary, secondary, and higher education reflects concerns that United States students are less proficient in these areas than are students in other countries. Additionally, there is a belief that mastery in STEM fields is now essential in order to join a highly technical and specialized workforce. However, not everyone is convinced that a STEM-focused curriculum is necessary and/or effective.

In your response you should do the following:

  • Respond to the prompt with a thesis that presents a defensible
  • Select and use evidence from at least three of the provided sources to support your line of Indicate clearly the sources used through direct quotation, paraphrase, or summary. Sources may be cited as Source A, Source B, etc., or by using the description in parentheses.
  • Explain how the evidence supports your line of
  • Use appropriate grammar and punctuation in communicating your

How to Approach the Question

Maybe your first thought upon seeing this block of text is to feel overwhelmed. But don’t panic. There are effective ways to approach the question so you will be more prepared in your response.

It’s a good strategy to first isolate the thesis . What is the main idea of the text, and what is its argument?

Try it out. Reread the prompt and see if you can identify what the statement is asking you to develop an opinion on.

Think you’ve got it? In this example, we will be focusing on whether or not a STEM-focused curriculum in K-12 education is necessary and/or effective. In short, we will be arguing either for (highlighting the benefits) or against (highlighting the pitfalls) a STEM-focused curriculum.

How do we know what this statement is asking us?

Well, the statement provides a lot of background information. For example, we receive a definition of what STEM stands for. As well, we know that since 2000, there has been a greater initiative for STEM-focused classes.

When you read the prompt for the first time, it’s a great strategy to learn how to differentiate between background and contextual information from the heart of the argument .

A good way to learn how to isolate the argument is to look for transition words. Usually, these appear near the end of the question. Words like “however” and “yet” are signals that the statement is offering a differing opinion. Typically, the statement will tell you which two positions it’s offering for argument. These opinions are usually signaled by contrasting transition words.

So, now that we know what the question is asking us, what is the best way to respond?

Synthesis Essay AP Lang Examples – Sample Answer

The following is an essay response I crafted to the above question. After reading the sample, I will break down what it does well and what areas can be improved.

A STEM-focused curriculum is not as essential to providing a meaningful K-12 education. Because the majority of high school students are not proficient in STEM-focused classes, prioritizing these classes causes harm to student’s mental health and academic performance.

As seen in Source A, 60% of high school seniors in the Midwest only scored a C average in math and science-based classes (Langston). This statistic suggests that the majority of students do not resonate with STEM classes and therefore perform poorly. Earning a low score in any class does not bode well for students’ mental health.

When looking at the primary argument in Source C, it’s clear that most high schoolers prefer creative outlets to fact-based research (Kohler). Allowing students the opportunity to be more creative and initiate conversations about coursework lets students be more active in their learning. When students can discuss the nuance in their opinions, more personal growth happens. These conversations are not always easy to have in STEM-focused classes.

As well, when looking back to Source A, it’s clear that high school students in the Midwest earned higher grades, on average, in their English and art classes (Langston). This figure suggests that students perform better in these classes because they relate more to the source material. When relating to what they learn, they perform better in class.

In conclusion, STEM-focused curriculum is not as essential in K-12 education because most high school students do not relate to their STEM classes. When students do not earn satisfactory grades in these classes, it negatively affects their future college applications and job prospects.

Synthesis Essay AP Lang Examples – Answer Breakdown

So, what does this essay response get right, and where can it be improved? Let’s start with what the response does well.

First, the response establishes its thesis right away. Usually, it’s a good idea to clearly state your argument within the first paragraph. Not only is this a good practice because a reader can easily identify your stance, but also you can refer to your thesis as you write to make you stay on track.

With your thesis, it’s also a good idea to include one to two supporting sentences with the reasons why the thesis is concluded . Like in this example, I wrote that STEM-focused classes should not be prioritized because they can negatively affect both mental health and academic performance.

Another positive aspect of this response is that it is sure to not only reference but also cites its sources . It’s important that the reader understand where your information is coming from. That way, the readers can ensure you are interpreting the sources correctly.

AP Lang Synthesis Essay (Continued)

However, when rereading the instructions, it’s clear that this response fails the basic requirement of referring to at least three sources. Always make sure to reread the instructions to ensure you meet the standard requirements for incorporating source material.

Further, this AP Lang synthesis essay does not fully support its arguments . Ideas are simply stated and are not expanded upon.

For example, I mentioned a few times that earning low grades in STEM classes leads to negative mental health for high school students. However, there is no source referenced that either confirms or denies this claim. Therefore, there is no sufficient evidence to support my argument. It relies purely on inference.

Additionally, this AP Lang synthesis essay does not arrive at a sufficient level of sophistication of thought . Basically, sophistication of thought means avoiding broad generalizations and vague claims. The more specific you can be, the better your argument will sound.

Synthesis Essay AP Lang – In Conclusion

In the end, it’s always helpful to read the prompt thoroughly before writing. As well, making notes while you read could be a good strategy to pinpoint main ideas both in the prompt and the sources. That way, you can reread the material quickly. Similarly, sketching an outline may also be helpful. In addition, you should always carefully read the instructions to ensure all guidelines are followed.

As long as you avoid broad generalizations and use enough supporting evidence for your claim, you will be on the right path!

  • High School Success

' src=

Meghan Dairaghi

With a BA in English and an MFA in Creative Writing, Meghan has served as a writing tutor at the University of Missouri St. Louis and Maryville University. Additionally, Meghan has held editorial roles at River Styx and Boulevard, and was a prose reader at Farside Review . Most recently, her work has been featured in Belle Ombre , Flypaper Lit , and Mag 20/20 , among others, and she was nominated for the Mary Troy Prize in Fiction. 

  • 2-Year Colleges
  • Application Strategies
  • Best Colleges by Major
  • Best Colleges by State
  • Big Picture
  • Career & Personality Assessment
  • College Essay
  • College Search/Knowledge
  • College Success
  • Costs & Financial Aid
  • Dental School Admissions
  • Extracurricular Activities
  • Graduate School Admissions
  • High Schools
  • Law School Admissions
  • Medical School Admissions
  • Navigating the Admissions Process
  • Online Learning
  • Private High School Spotlight
  • Summer Program Spotlight
  • Summer Programs
  • Test Prep Provider Spotlight

College Transitions Sidebar Block Image

“Innovative and invaluable…use this book as your college lifeline.”

— Lynn O'Shaughnessy

Nationally Recognized College Expert

College Planning in Your Inbox

Join our information-packed monthly newsletter.

I am a... Student Student Parent Counselor Educator Other First Name Last Name Email Address Zip Code Area of Interest Business Computer Science Engineering Fine/Performing Arts Humanities Mathematics STEM Pre-Med Psychology Social Studies/Sciences Submit

synthesis ap lang format

How to Write the AP Lang Synthesis Essay

synthesis ap lang format

AP Lang test is the logical conclusion to the introductory college English composition course. And its most important (and often difficult) part is the AP Lang synthesis essay. Despite it being the very basic layer of your future composition skills, it’s a very complicated challenge to approach unprepared. Besides, it's details may change year to year. So let’s have a look with our coursework writing services team at what your AP Lang exam 2022 might look like.

What is AP Lang?

AP Lang is a relatively lengthy test. There are several AP rubrics that a student must be well-versed in to hope to pass it. The first section includes reading and writing, while the second is slightly more freeform and includes three different types of essays.

Among those three, the most interesting and, coincidentally, oftentimes the hardest to deal with is the AP Lang synthesis essay rubric. Today will focus on it specifically to make sure you know exactly what you’re going to be facing during your test.

What Is a Synthesis Essay AP Lang?

At its core, the AP Lang synthesis essay is a pretty straightforward part of the AP Lang test. It might look pretty similar to the reading section of the exam. However, simply finding the right information isn’t enough. When writing a synthesis essay, you should not only gather the data but also distill it into your personal opinion.

This fine line may seem difficult to spot, but it is there. And it’s that small difference that can make or break your exam run. So try to follow the steps one by one and not lose focus. Writing a good synthesis essay is as easy as following the rules. If you feel this task is too difficult for you, you can leave us your ' write an essay for me ' request and we will do it for you.

AP Lang Synthesis Essay Outline

Looking through AP Lang essay examples, you might notice that the overall structure doesn’t really differ too much from your standard essay outline. You have your introduction, your body, and your conclusion. But the important thing to note is where your arguments are supposed to come from.

You’re not supposed to just go off on a rant. The task requires you to base your supporting evidence on at least three sources. And you will have to ensure your essay has solid roots. Here’s what a basic AP Lang exam synthesis essay outline should look like:

  • Introduction

Provide sufficient context for the topic you are about to cover. You can do a quick overview of prevailing opinions you have grasped while browsing through your source materials.

Write a short and compelling thesis statement. This will be your ground zero for the rest of the essay. So make sure it reflects your opinion. What is a thesis statement you can read in our special article.

  • Body Paragraphs

Dedicate at least one paragraph to every source you’re using. Start with presenting the evidence you have gathered from that source and go on to explain how it formed your opinion on the topic and why it should be considered.

Quickly go through your line of reasoning and reinforce what you have already covered. Finish up with restating your thesis as you’re supposed to logically arrive at it after all the evidence you have presented. That’s how you write a conclusion properly.

Different Forms and Types of Synthesis Essay: Explanatory vs. Argumentative Synthesis Essays

When it comes to writing a synthesis essay AP Lang, there are several types of essays you should consider. The most common ones are the AP Lang argument essay and explanatory essay. The clues as to how each of them should look are hidden within their names but let’s go over them to clear any confusion.

An explanatory essay’s goal is to go over a certain topic, discuss it in detail, and ultimately show a high level of understanding of the said topic. You don’t necessarily have to get into a heated argument with the reader trying to convince them of something. All you need to do is create an impartial overview.

On the other hand, an argumentative essay has to do with personal opinions. And while there is a time and a place for bias, it still has to be as impartial and factual as possible. When proving your point, try not to devolve into emotional arguments but stick to logic and cold truths. This will make your argument way more solid.

Synthesis Essay Structure

In the general case, you don’t really need to look for a synthesis essay AP Lang example to get a solid grasp on how its structure should look like. You can safely fall back on your high school essay writing knowledge, and you’ll be mostly safe.

What you should pay attention to is your writing style and content. A synthesis essay is identified less by its structure and more by the way you form and present your arguments to the reader. It’s when you get a specific essay type (like an argumentative essay) that you should pay attention to slight changes in format.

Argument Essay Structure

The best way to understand argumentative essay structure is to study any well-written AP Lang argument essay example. Standard AP Lang essays have very distinctive features that are very easy to spot and emulate. They follow a very rigid form and employ specific rhetorical devices that you’ll be able to pick up after you analyze them once or twice quickly.

How Many Paragraphs Should an AP Lang Synthesis Essay Be?

The number of paragraphs in an AP Lang synthesis essay can indeed make a difference. Your arguments should be concise and pointed. Spreading them out throughout many paragraphs may seem like a good idea to fill in the space. But it’s actually detrimental to your final score. You can get a basic understanding of what your score is going to be using an AP Lang score calculator.

The same goes for too few paragraphs. Don’t even try to squeeze your entire line of thought into a single body paragraph. Generally, the minimum number of sources you should address is three. Any less, and you are getting a lower score. So try to keep it somewhere in the middle. Three to five body paragraphs is an optimal number. Don’t forget to add an intro and a conclusion to it and you’re all set. A well-written essay has a clear and easily identifiable structure.

How to Write AP Lang Synthesis Essay: Guide

How to Write the AP Lang Synthesis Essay

In order to write a decent essay, all you have to do is follow these simple steps. Performing a rhetorical analysis essay example, AP Lang won’t give you insight into how it was built from the ground up. But looking at this list might.

Step 1. Read the Prompt

It may sound like a no-brainer. But it’s actually more important than you can imagine. Don’t skip right past this step. It’s very easy to misunderstand the task under stress. And if you do slip up in the beginning - the entirety of your work after that is wasted.

Step 2. Analyze the Sources Carefully

The same goes for your sources. Take your time reading them. Try to spot every smallest detail, as even a single one can help you better incorporate your evidence into the body of your essay. You can begin outlining the general points of your essay in your head at this point.

Step 3. Come Up with a Strong Thesis Statement

Your thesis statement is the baseline of your writing. Make it short and clear. Try not to overthink it too much.

Step 4. Fill in Your Essay Outline

Start filling out your outline step by step. You don’t have to go from top to bottom. If you feel like you’re struggling - skip to the next part and return to the problem paragraph later. The use of rhetorical devices AP Lang is also pretty important. So once you flesh out your essay a bit, spend some time trying to come up with the perfect wording.

Step 5. Finalize

The first finished version of your essay is a draft. Don’t be hasty to turn it in. Read over it a couple of times. Make sure everything is in order. You can switch some of the parts around or rewrite some sections if you have the time. Ideally, at this stage you should have enough time to eliminate all grammatical errors that may still be present in your essay. Polish it to perfection.

Useful Tips

Here are some useful tips that might make the writing process a bit easier for you:

  • Use either APA or Chicago style to cite your sources
  • Have a schedule to understand how much time you have for each section
  • Leave as much time as you can for editing and proofreading
  • You can never over study the source material. Spend as much time as you can reading into it
  • Don’t linger on the surface of your essay subject. Dive in and show your complex understanding of the material
  • Avoid using private life anecdotes to support your case unless the essay type specifically allows it. These don’t make for a convincing argument.
  • Use as many supporting arguments as you can but make sure they are actually solid and relevant to your thesis
  • Check with your thesis from time to time. The entirety of your text should align with it

Need help with academic deadlines?

Falling back on your deadlines? Use our term paper writing services to relieve you while you get back on your feet.

AP Lang Essay Prompts

Here are some interesting prompts. Some of them could be found in the previous iterations of the test; you may have spotted them in some of the AP Lang essay examples. Others are there to help you practice for the AP Lang exam 2022.

  • The John F. Kennedy Presidential Library and Museum, dedicated in 1979, was founded in memory of the president and contained archives pertaining to his administration. On June 24, 1985, then President Ronald Reagan joined members of the Kennedy family at a fundraising event to help the Kennedy Library Foundation create an endowment to fund and support the presidential library. The following is an excerpt from the speech Reagan gave at that event. Read the passage carefully. Write an essay that analyzes the rhetorical choices Reagan makes to achieve his purpose of paying tribute to John F. Kennedy.
  • On August 29, 2009, then-President Barack Obama delivered a eulogy at the funeral of Senator Ted Kennedy in Boston, Massachusetts. Kennedy served in the United States Senate from 1962 until his death. Obama served with him in the Senate from 2005 until Obama was elected president in 2008. The following is an excerpt from Obama’s speech. Read the passage carefully. Write an essay that analyzes the rhetorical choices Obama makes to achieve his purpose of praising and memorializing Kennedy.
  • On April 9, 1964, Claudia “Lady Bird” Johnson, who was at the time the First Lady of the United States, gave the following speech at the first-anniversary luncheon of the Eleanor Roosevelt Memorial Foundation. The foundation is a nonprofit division of the Franklin D. Roosevelt Presidential Library dedicated to the works of former First Lady Eleanor Roosevelt, who passed away in 1962. Read the passage carefully. Write an essay that analyzes the rhetorical choices Johnson makes to achieve her purpose of paying tribute to Eleanor Roosevelt.

In your response, you should do the following:

• Respond to the prompt with a thesis that analyzes the writer’s rhetorical choices.

• Select and use evidence to support your line of reasoning.

• Explain how the evidence supports your line of reasoning.

• Demonstrate an understanding of the rhetorical situation.

• Use appropriate grammar and punctuation in communicating your argument.

AP Lang Essay Example

Here is a decent if a bit shortened, AP Lang rhetorical analysis essay example you can use for reference.

Literature to Prepare for AP Lang

How to Write the AP Lang Synthesis Essay

And here is a list of some great AP Lang books that will help you prepare for the exam. Not all of them are immediately useful, but most will help you enhance your writing and analytical abilities to get a better score in the end.

  • The Odyssey
  • Don Quixote
  • A Midsummer Night's Dream
  • Pride and Prejudice
  • Wuthering Heights
  • Oliver Twist
  • Crime and Punishment
  • Adventures of Huckleberry Finn

If you have thoughts of "who could do my paper for me," do not forget that you can contact us. Or, if you have a finished paper and you need to make edits to it, leave us a ' rewrite my essay ' request and we will do it as soon as possible.

Related Articles

Types of Narrative Writing

logo-type-white

AP® English Language

How to ace the ap® english language and composition synthesis essay.

  • The Albert Team
  • Last Updated On: March 1, 2022

how_to_ace_the_ap_english english language synthesis essay

The newest section of the AP® English Language and Composition Exam, the synthesis essay, is one of three essays you will be completing during the examination’s 2-hour free-response period. However, you’ll also have a 15-minute reading and planning period just for this essay, and if you use this time to plan effectively, you can’t go wrong.

Before we get into specific advice on how to handle the AP® English Language and Composition synthesis essay, you need to know what this part of the test really is. It is very similar to the argumentative essay you will also write as part of this exam, except that you are provided with a wealth of source material from which to draw some support for your ideas.

While this in some ways makes the AP® English Language and Composition synthesis essay easier than the argument essay (because you can use quotations, point to authoritative sources for support, etc.), there is an extra element of complexity, and the AP® readers want to see how well you can sort through your source material and put it to good use – which makes planning all that much more important. This brings us to our first tip…

1. Use Your 15-Minute Planning Period Wisely.

The main purpose of this 15-minute period is to give you time to read the source materials. This essay will present you with several sources providing different information about or opinions on a certain topic. Make sure you don’t just skim them, but read them closely – make notes, underline key sections you may want to quote later, etc.

You should also begin outlining your essay and considering your opinion on the subject; have this opinion in mind before you start writing the essay, as you will use it to construct your thesis.

You’ve already learned how to structure persuasive essays in this class and in other classes you have taken; put that knowledge to good use now, and have your main points set out before you start writing. Try to have a thesis statement written by the time you start the essay – your thesis should establish your opinion and the general reasons you feel this way; the rest of your essay will go on to justify and exemplify these reasons. Also write down some of the main points upon which you will base subsequent paragraphs and mark quotes or sections of the sources you can use in each of these paragraphs.

2. Evaluate Your Sources.

ap exams score

Every source you can use for the AP® Language and Composition synthesis essay will have a small box above it explaining where it comes from and who said it – to see exactly what this looks like, check out the free synthesis essay sample questions at AP® Central. There are also public sample questions available there for the rest of the AP® English and Composition Exam .

Keep all information about your sources in mind when you’re quoting them or using them to support your arguments. What journal an article appeared in can say a great deal about its potential biases. For example, consider a question on the environmental impacts of corporate practices – an environmental journal is obviously going to be biased in favor of more environmental regulation, while a report from a company spokesperson will probably gloss over some of the negative impacts of his company. Think critically.

3. Keep Your Tone Consistent.

There is no hard-and-fast advice about what tone you should take – some students try to inject a little humor into their essays while others prefer to be as serious as possible, some are extremely critical and others more accepting. However, the one thing you really have to do while writing the AP® Language and Composition synthesis essay (or any other essay) is keep your tone consistent. Jot some tone-related ideas down as you outline during the 15-minute reading period, and keep in mind everything you’ve learned about tone and other aspects of rhetoric so far this year.

4. Use Rhetorical Technique to Your Advantage!

The various rhetorical practices you’ve been learning about all year can be put to good use here. This class and this test aren’t just about recognizing and analyzing these techniques when others use them, but about preparing you for college and your career by teaching you how to use them effectively yourself. However, this isn’t just about writing a beautiful essay, so read on to Tip # 5!

5. Your Argument Must be Well-Crafted.

The AP® English Language and Composition Exam synthesis essay does not have right or wrong answers; rather, it asks you for your opinion. The AP® Examiner cannot take points off because she disagrees with you. However, you must show logical basis for your opinion, drawing on both the sources AND your own knowledge and experience.

To do this, make sure you have a clear and complete thesis. Make sure the ideas expressed in the beginning of each paragraph or section support the thesis, and that you in turn show how those ideas are supported by a source or through your own knowledge and experience. Don’t generalize or write anything down that you can’t support.

Looking for AP® English Language practice?

Kickstart your AP® English Language prep with Albert. Start your AP® exam prep today .

Interested in a school license?​

Popular posts.

AP® Physics I score calculator

AP® Score Calculators

Simulate how different MCQ and FRQ scores translate into AP® scores

synthesis ap lang format

AP® Review Guides

The ultimate review guides for AP® subjects to help you plan and structure your prep.

synthesis ap lang format

Core Subject Review Guides

Review the most important topics in Physics and Algebra 1 .

synthesis ap lang format

SAT® Score Calculator

See how scores on each section impacts your overall SAT® score

synthesis ap lang format

ACT® Score Calculator

See how scores on each section impacts your overall ACT® score

synthesis ap lang format

Grammar Review Hub

Comprehensive review of grammar skills

synthesis ap lang format

AP® Posters

Download updated posters summarizing the main topics and structure for each AP® exam.

Interested in a school license?

synthesis ap lang format

Bring Albert to your school and empower all teachers with the world's best question bank for: ➜ SAT® & ACT® ➜ AP® ➜ ELA, Math, Science, & Social Studies aligned to state standards ➜ State assessments Options for teachers, schools, and districts.

pep

Find what you need to study

2024 AP English Language and Composition Exam Guide

12 min read • august 18, 2023

A Q

Your guide to the 2024 AP English Language and Composition exam

We know that studying for your AP exams can be stressful, but Fiveable has your back! We created a study plan to help you crush your AP English Language and Composition exam. This guide will continue to update with information about the 2024 exams, as well as helpful resources to help you do your best on test day.  Unlock Cram Mode  for access to our cram events—students who have successfully passed their AP exams will answer your questions and guide your last-minute studying LIVE! And don't miss out on unlimited access to our database of thousands of practice questions. FYI, something cool is coming your way Fall 2023! 👀

Format of the 2024 AP English Language and Composition exam

This year, all AP exams will cover all units and essay types. The 2024 AP English Language and Composition exam format will be:

Section I: Multiple Choice - 45% of your score

45 questions in 1 hour

Section II: Free Response Section - 55% of your score

2 hours and 15 minutes for:

1 synthesis essay

1 rhetorical analysis essay

1 argument essay

Scoring Rubric for the 2024 AP Lang Essays

Synthesis Essay

1 point for a defensible thesis that responds to the prompt

Evidence and Commentary

Max of 4 points for providing evidence from at least 3 sources that support the line of reasoning AND commentary that explains and analyzes the evidence

Sophistication

1 point any of the following:

Creating a nuanced argument

Showing the limitations of the argument

Making effective rhetorical choices

Employing a style that is vivid and persuasive

Rhetorical Analysis Essay

1 point for a defensible thesis that analyzes rhetorical choices

Max of 4 points for providing specific evidence AND consistently explaining how the evidence relates to the line of reasoning AND showing how the rhetorical choices contribute to the author's message .

1 point for any of the following:

Explaining the significance of the rhetorical choices ( rhetorical situation )

Explaining the complexities of the passage and their purpose

Argument Essay

1 point for a defensible thesis

Max of 4 points for providing specific evidence AND consistently explaining the relevance of that evidence .

Crafting a nuanced argument by identifying complexities

Explaining the limitations of the argument by placing it in a broader context

Making rhetorical choices to improve the argument

Check out our study plan below to find resources and tools to prepare for your AP English Language and Composition exam.

When is the 2024 AP English Language and Composition Exam and How Do I Take It?

How should i prepare for the ap lang exam.

First, take stock of your progress in the course so far. What areas have you excelled and which sections need more focus? Download the AP English Language Cheatsheet PDF - a single sheet that covers everything you need to know at a high level. Take note of your strengths and weaknesses!

Build your study plan to review every unit and question type, but focus most on the areas that need the most improvement and practice. We’ve put together this plan to help you study between now and May. This will cover all of the units and essay types to prepare you for your exam

Practice essays are your best friends! The more essays you write, the more automatic the process will come, and the easier the AP exam will be!

Try some of the past exam questions here

We've put together the study plan found below to help you study between now and May. This will cover all of the units and essay types to prepare you for your exam. Pay special attention to the units that you need the most improvement in.

Study, practice, and review for test day with other students during our live cram sessions via  Cram Mode . Cram live streams will teach, review, and practice important topics from AP courses, college admission tests, and college admission topics. These streams are hosted by experienced students who know what you need to succeed.

Pre-Work: Set Up Your Study Environment

Before you begin studying, take some time to get organized.

🖥 Create a study space.

Make sure you have a designated place at home to study. Somewhere you can keep all of your materials, where you can focus on learning, and where you are comfortable. Spend some time prepping the space with everything you need and you can even let others in the family know that this is your study space. 

📚 Organize your study materials.

Get your notebook, textbook, prep books, or whatever other physical materials you have. Also, create a space for you to keep track of review. Start a new section in your notebook to take notes or start a Google Doc to keep track of your notes. Get yourself set up!

📅 Plan designated times for studying.

The hardest part about studying from home is sticking to a routine. Decide on one hour every day that you can dedicate to studying. This can be any time of the day, whatever works best for you. Set a timer on your phone for that time and really try to stick to it. The routine will help you stay on track.

🏆 Decide on an accountability plan.

How will you hold yourself accountable to this study plan? You may or may not have a teacher or rules set up to help you stay on track, so you need to set some for yourself. First, set your goal. This could be studying for x number of hours or getting through a unit. Then, create a reward for yourself. If you reach your goal, then x. This will help stay focused!

🤝 Get support from your peers.  

There are thousands of students all over the world who are preparing for their AP exams just like you! Join  Rooms  🤝 to chat, ask questions, and meet other students who are also studying for the spring exams. You can even build study groups and review material together! 

2024 AP Lang Study Guide

🚧 unit 1 foundations of rhetoric: analysis of the rhetorical situation and claims ., big takeaways:.

Unit 1 is an introductory unit that lays the foundations for the reading skills associated with how to understand and analyze complex texts. Skills here include identifying the ASPECTS of a text, analyzing the claim given and the evidence used to support that claim, and determining the function of the “chunks” in the argument. Because the content in this unit is very foundational, it is looped throughout the rest of the course instruction.

Definitely do this:

📚 Read these study guides:

Unit 1 Overview: Claims , Reasoning , and Evidence

1.1 Identifying the purpose and intended audience of a text

1.2 Examining how evidence supports a claim

1.3 Developing paragraphs as part of an effective argument

🎥 Watch these videos:

College Board’s Instructional Video: Overview of The Rhetorical Situation .

Fiveable’s How to Read Like an AP Student .

Rhetorical Analysis Thesis Statements  

Rhetorical Analysis Body Paragraphs

✍️ Practice:

Use the Fiveable ASPECTS Guidesheet to help you break down a complex text.

🗺 Can you identify these rhetorical devices?

You won’t be asked to name drop on the exam, but it can be helpful to use devices when discussing strategies. Try this Quizlet to help prepare.

Unit 2 Foundations of Argument: Analysis of an author’s choices in appeals and evidence

Unit 2 is an introductory unit that builds onto the foundations of rhetorical ASPECTS and moves toward planning and writing your own arguments. This unit focuses on the relationships between subject, speaker, and message, including examination of the structure and purpose of the given argument. The unit then moves into the developing thesis statements and building your own arguments with a clear line of reasoning .

Unit 2 Overview: Organizing Information for a Specific Audience

2.1 Analyzing audience and its relationship to the purpose of an argument

2.2 Building an argument with relevant and strategic evidence

2.3 Developing thesis statements

2.4 Developing structure and integrating evidence to reflect a line of reasoning

College Board’s Instructional Video: Identify Rhetorical Situation in a Pre 20th Century Text .

Fiveable’s video on How to Find Rhetorical Devices  

📰 Check out these articles:

Here’s a list of recommended rhetorical devices with definitions and examples!

Use the Fiveable Rhetorical Precis Guidesheet to help you break down a complex text.

🗺 Can you identify these elements of practical argument?

You won’t be asked to name drop of the exam, but it can be helpful to use devices when discussing strategies. Try this Quizlet to help prepare.

👥 Unit 3 Confluence: Synthesis of multiple sources in argumentation

Unit 3 approaches multiple perspectives in argument through the lens of synthesis (that’s FRQ 1). In this study, you learn to identify effective and faulty reasoning while integrating a variety of evidence from credible resources that is properly cited in an original text.

Unit 3 Overview: Perspectives and How Arguments Relate

3.1 Interpreting character description and perspective

3.2 Identifying and avoiding flawed lines of reasoning

3.3 Introducing and integrating sources and evidence

3.4 Using sufficient evidence for an argument

3.5 Attributing and citing references

3.6 Developing parts of a text with cause-effect and narrative methods

Fiveable’s Introduction into Synthesis Essays and How to Begin Your Argument

College Board’s Instructional Video: Complexity in Argument .

🗺 Can you identify these elements of synthesis?

👀 Unit 4 Reasoning : Analysis of argument from introduction to conclusion

Unit 4 includes a greater depth of focus on the writing of effective arguments -- the line of reasoning created in the introduction, built with modes of discourse, and strengthened in the conclusion. An important note about these skills of argumentation is that they build toward all parts of every FRQ. 

Unit 4 Overview: How writers develop arguments, intros, and conclusion

4.1 Developing and connecting thesis statements and lines of reasoning

4.2 Developing introductions and conclusions

4.3 Adjusting an argument to address new evidence

College Board’s Instructional Video: Understanding a Line of Reasoning .

Fiveable’s Effective Annotations .

Try Fiveable’s Guide to LOR Body Paragraphs .

🗺 Can you identify the rhetorical modes?

You won’t be asked to name drop them on the exam, but it can be helpful to use devices when discussing strategies. Try this Quizlet to help prepare.

🧐 Unit 5 Commentary and Analysis: Analysis of complex argument and intentional rhetoric

In Unit 5, the skills look at the minutiae involved in argumentation: development of the line of reasoning that produces strong commentary and maintains the primary claim through all parts of the writing. To achieve these goals, this unit includes a focus on transitions , modifiers , and qualifications for argumentative perspective .  

Unit 5 Overview

5.1 Maintaining ideas throughout an argument

5.2 Developing commentary throughout paragraphs

5.3 Using modifiers to qualify an argument and convey perspective

5.4 Using transitions

Fiveable’s video on How to Improve Analysis Part 1 and Part 2

As well as how to Embed Quotes into Body Paragraphs  

Rhetorical Analysis Body Paragraphs  

Synthesis Essay Body Paragraphs  

Argument Essay Body Paragraphs

Tara Seale’s adaptation for Creating a Line of Reasoning .

🏃‍♂️ Unit 6 Rhetorical Risks: Analysis of multiple perspectives , bias , and shifts with new evidence

In Unit 6, you will notice a direct link building on the ideas of Unit 3 as this instruction looks at position and perspectives while synthesizing information strategically to support a claim.  For greater depth, this unit moves to modify a current argument to include new evidence .

Unit 6 Overview: Position, Perspective , and Bias

6.1 Incorporating multiple perspectives strategically into an argument

6.2 Recognizing and accounting for bias

6.3 Adjusting an argument to new evidence

6.4 Analyzing tone and shifts in tone

College Board’s Instructional Video: Creating a Nuanced Argument .

Fiveable’s video on Tracking an Author’s Argument  

🚀 Unit 7 Complex Argumentation: Analysis of effective arguments, including concession and refutation

The skills of Unit 7 are about putting all units of study together to look at the complexity of a given argument and the effectiveness of the pieces built into that argument.  Though many teachers will have addressed counterarguments, concessions, and refutations before reaching this unit, those skills are highly scrutinized in this segment of learning.

Unit 7 Overview: Successful and Unsuccessful Arguments

7.1 Examining complexities in issues

7.2 Considering how words, phrases, and clauses can modify and limit an argument

7.3 Examining how counterargument or alternative perspectives affect an argument

7.4 Exploring how sentence development affects an argument

Fiveable’s video on Arguments and Counterarguments  

College Board’s Instructional Video: How Argument Demonstrates Understanding .

Check your progress with Fiveable’s AP Language Skills Matrix .

📝 Unit 8 Style: Analysis of how style influences the audience movement

Unit 8 covers how to understand the influence style has on the audience , and the purpose behind each decision. By analyzing these various tactics, students are able to understand the author’s audience , and how to effectively persuade them. Style is an important part in connecting the rest of the course and understanding how the rhetorical choices and devices are used to accomplish a purpose .

Unit 8 Overview: Stylistic Choices

8.1 Choosing comparisons based on an audience

8.2 Considering how sentence development and word choice affect how the writer is perceived by an audience

8.3 Considering how all choices made in an argument affect the audience

8.4 Considering how style affects an argument

Fiveable’s Analysis of the Mindset of the Audience

College Board’s Instructional video: Analyzing and Understanding the Audience

College Board’s explanation of Elements and Context for Style  

Review this quizlet on Elements of Style for more practice.

✏️ Unit 9 Craft: Creation of your own complex argument with synthesis and rhetoric

The final unit of AP Language and Composition covers how to effectively form your own arguments by acknowledging and understanding complexities to create a nuanced and sophisticated argument. It focuses on your ability to comprehend and connect multiple sources to create a well reasoned, and detailed argument as well as how to add in your own rhetorical devices and choices to make your writing more persuasive and effective.

Unit 9 Overview: Developing a Complex Argument

9.1 Strategically conceding, rebutting, or refuting information

9.2 Crafting an argument through stylistic choices like word choice and description

Fiveable’s video on Creating your own Synthesis Arguments

College Board’s video on Complexities within Arguments and How to Create a Nuanced Argument

Key Terms to Review ( 38 )

Argument Structure

Author's Message

Cause-Effect Method

Comparisons

Conclusions

Counterargument

Introductions

Line of Reasoning

Multiple Perspectives

Narrative Method

Objective Reasoning

Perspective

Qualifications

Rhetorical Choices

Rhetorical Situation

Sentence Development

Stylistic Choices

Subjective Reasoning

Textual Evidence

Thesis Development

Thesis Statement

Tone Shifts

Transitions

Word Choice

Fiveable

Stay Connected

© 2024 Fiveable Inc. All rights reserved.

AP® and SAT® are trademarks registered by the College Board, which is not affiliated with, and does not endorse this website.

How to Write a Synthesis Essay AP Lang | Guide to Getting a Perfect Score

The AP Language and Composition exam is notoriously difficult. Even top, A+ language arts students have found it exceptionally challenging. 

the ap language and composition exam is notoriously difficult

Beyond diligently studying with a  top-rated AP Lang review book , you should also carefully review our comprehensive guide on how to write a synthesis essay for AP Lang featured in this article.

Once you complete the supremely humbling hour-long multiple-choice section, making up only 45% of your score, the trial has only just begun. Now, already fatigued from the first section, you must begin the grueling, two-hour free response section of the exam, which accounts for a whopping 55% of your final exam score. This section requires you to compose three essays of distinct types: an argumentative essay, a rhetorical analysis essay ( you also need to learn how to use AP Lang rhetorical devices ) and, the most dreaded of all, the synthesis essay.

Luckily, there are some great options to help you prepare to nail your synthesis essay. So, read on for everything you need to know to knock your synthesis essay out of the park!

How to Write Synthesis Essays AP Lang

One of the most challenging aspects of the AP synthesis essay is figuring out what the prompt is asking you to do. After all, what does it mean to “synthesize” something anyway? And what’s the difference between a synthesis essay and an evidence-based argumentative essay?

In this guide, we’ll answer these questions and a lot more. We’ll help you establish a solid understanding of what synthesis is, how to do it, and how to use it to write a high-scoring essay, according to College Board’s AP Language and Composition scoring guidelines . Lastly, we’ll leave you with some advice about things to be sure and attend to in your essay, as well as the most important things to avoid. 

Introduction to Syntheses

Before we get into all the details and advice on how to write a quality synthesis essay, we first need to answer a critical question: what is a synthesis essay?

Simply put, a synthesis essay is a piece of writing that brings together information and ideas from two or more sources. 

The synthesis part comes in as you begin to develop connections between the sources

The synthesis part comes in as you begin to develop connections between the sources, whether they are in agreement, disagreement, approach the same topic from different angles, or simply provide ideas on different topics that can in some way contribute some other discourse.

This, of course, is just a very basic introduction to what a synthesis essay is.

Throughout this article, the concept will surely become clearer to you.

Two Types of Syntheses

There are essentially two types of synthesis essay that you will run into: argumentative syntheses and explanatory syntheses. Luckily, you really only need to master one of them, the argumentative synthesis, as this is the most common expectation on AP exams.

Explanatory Synthesis

An explanatory synthesis essay is exactly what it sounds like. It the type of writing in which you will be asked to explain the arguments and information presented in your sources. You should also seek out connections and contrasting elements between the sources in order to give your essay a certain level of nuance and to display your deeper understanding and reasoning skills. 

Most of the time, AP Language and Composition exams won’t focus on explanatory synthesis essays. However, it would be wise to use some explanatory techniques even within your argumentative synthesis essay.

Argumentative Synthesis

When someone asks, “what is a synthesis in writing?” they’re typically referring to argumentative synthesis, and this is especially the case for the AP exam. 

At its most basic, an argumentative synthesis essay is on in which you must present your own opinions and support them with appropriate ideas and information from your sources. Most importantly, the thesis for your argumentative synthesis essay must be a proposition that can be debated. That is, there must be another potential argument against your own. 

Standards for Synthesis Essays

unfortunately, these prompt-specific rubrics are not available to the public until after the exams

The people who will be scoring your AP Lang synthesis essay use a very clearly defined rubric to determine your score according to various criteria. Unfortunately, these prompt-specific rubrics are not available to the public until after the exams.

That said, we can still gain a wealth of useful information from past scoring guidelines.

Here are some of the elements of high-scoring essays that seem to be fairly constant from exam to exam, regardless of the essay prompt topic:

Supply useful context on the topic

Give a sense of why the topic is important

Engage with the complexity of the subject

Foreground your opinion on the topic

Offer thorough and thoughtful analysis of quotations, paraphrases, etc.

Synthesize source material by finding connections with your own ideas and opinions

Properly attributes ideas to sources

Conclude with more than just a summary by answering the “so what?” question

Techniques for Developing Synthesis Essays

Before you can start writing a quality synthesis essay, you need to spend some time developing your ideas and seeing how they do or do not relate to your source materials.

The following is a list of steps that you should always take before you start writing the bulk of your actual essay. These guidelines will be enormously helpful when it comes time to figure out what you want to say in your essay. If you read this carefully and take seriously these suggestions, you’ll have no trouble coming up with interesting and complex ideas for your essay.

What’s your purpose? Before you do anything, you need to determine what the prompt is asking you to do. Obviously, it’s going to ask you to synthesize some stuff, but keep an eye out for these helpful guide words:

Compare/Contrast

Read the source material; then, read it again to annotate. Once you’ve read through the sources once, go back and reach each one again, this time with a pencil to underline and add notes as you go.

Formulate your thesis statement. After reading and taking notes on your sources, you’re ready to brainstorm your thesis statement. As you do this, try to keep track of potential aspects of each source that you can use to support your claim. 

Sketch an outline. When you have a thesis statement down (at least a tentative one—you should always be open to revising it as you go), you should sketch a simple outline that includes your thesis statement, supporting points that you can use for topic sentences, and a rough idea of how you will incorporate your sources. 

If you follow these steps closely, there’s no doubt that you’ll be ready to start writing what is sure to be a clever, thoughtful, and nuanced synthesis essay.

How to Write a Good Synthesis Essay?

as you go about the work of composing your essay, there are several approaches and strategies to can implement at different points in the essay

Now that we know what a synthesis essay is, what AP exam scorers will be looking for in your essay, and what techniques you can use to develop your essay topic, it’s time to look at some different strategies for how to actually write a synthesis essay in AP Lang.

As you go about the work of composing your essay, there are several approaches and strategies to can implement at different points in the essay.

None of these elements will be particularly effective in isolation, so be sure to use a variety of these strategies to enhance the complexity and depth of your argument.

Summarize. Summarizing ideas and source material is easy, and for that reason it is not going to do a whole lot in the way of getting you a great score. It is, however, an incredibly useful tool that, when used in conjunction with other strategies on this list, can be quite effective.

Compare and Contrast. This is typically the level of synthesis that most people start at after summarizing the content they are considering. It’s not exactly “high-order” analysis, but it can be very useful in establishing the positions of your sources and creating a foundation on which you can present your own ideas and opinions.

Give an Example. A great way to show you understand a certain concept is to apply your understanding through an example. You can think of examples that illustrate the concepts you’re dealing with as a way to clarify your topic and also to support your own arguments. 

“They Say, I Say.” This is the classic, and possibly most effective, synthesis move. After you’ve presented a quotation or paraphrase of one or more sources’ ideas, move on to explain your own position as it relates to theirs. It becomes much easier to state your own ideas and opinions when you do it within the context of the larger discussion of the topic.

Synthesis Essay Structure

One of the most sure-fire ways to earn a passing score on your synthesis essay is to ensure you structure it effectively. To do this well, it’s a smart idea, prior to writing your essay, to sketch out a quick outline of the essay’s structure. Your outline doesn’t need to be especially detailed, but it will be tremendously helpful for you to have a general plan to work from.

Another good idea that will help you learn how to structure your essay is by looking at a synthesis example.

Helpfully, the College Board’s AP Language and Composition exam site supplies several samp le essays for you to get an idea of what good, average, and poor synthesis essays look like.

if you take some time to read these examples, you’ll have a much better vision of what the scorers are looking for in terms of a well-structured synthesis essay

If you take some time to read these examples, you’ll have a much better vision of what the scorers are looking for in terms of a well-structured synthesis essay.

That said, there is a very basic structure for synthesis essays that, if followed carefully, will guarantee that your essay is, at the very least, structured logically. This format is essentially the standard format for any basic five-paragraph essay. Take a look.

Basic Essay Structure

Introduction Paragraph

Give some BACKGROUND information and CONTEXT on the general topic of your essay.

Briefly introduce the SOURCES that you will be using.

Present your THESIS STATEMENT (this is usually the very last sentence of the paragraph).

Body Paragraph

Start with a TOPIC SENTENCE that supports your thesis statement.

Cite one or more SOURCES that support your topic sentence.

Provide COMMENTARY and ANALYSIS on the ideas you have just cited from your sources. 

Use a TRANSITION WORD or PHRASE to guide the reader logically toward the ideas you will present in the topic sentence of the next paragraph.

Body Paragraphs 2-?

Repeat the steps in the previous section with different topic sentences and source references, all of which must support your thesis statement. 

Include as many body paragraphs as you have points and topic sentences to justify. Typically, a good AP Lang synthesis essay will have around 3-4 well-constructed and reasoned body paragraphs, but this is just a general guideline.

Conclusion Paragraph

Restate your THESIS statement in a new and interesting way (that is, do not simply repeat your thesis word-for-word as it appears in the introduction!).

Tie it all together by briefly summarizing your main points.

Answer the “SO WHAT?” QUESTION by explaining why your argument matters and what the implications of it might be. Try to broaden your scope in order to show the reader how it fits in with the “big picture.”

Synthesis Writing Dos and Don’ts

synthesis ap lang format

Finally, we want to leave you with a quick list of things to strive for and things to avoid at all costs. 

DO Develop a Strong, Clear Thesis Statement

DO Use Topic Sentences 

DO Cite Your Sources Accurately and Appropriately

DO Sketch a Basic Outline 

DO Pace Yourself

DO Proofread and Revise Your Essay Carefully

DON’T Overdo It with Summaries

DON’T Start Paragraphs with Quotations

DON’T Get Overwhelmed by the Sources

DON’T Use Other People’s Ideas without Citing Your Sources

DON’T Use Overly Lengthy Quotations or Paraphrases

Leonard Haggin

I created this site to help students like you learn from the experiences my team had learned during our extensive academic careers. I am now studying Law at Stanford, but I also make time to write articles here in order to help all you fellow students advance in your academic careers and beyond. I hope our efforts on Study Prep Lounge will arm you with the knowledge you need to overcome whatever trial or test you find in front of you.

Leave a Reply:

Save my name, email, and website in this browser for the next time I comment.

Automated page speed optimizations for fast site performance

Nerdpapers logo

We have sent you an email with a 6 digit code to:

Didn't receive an email? Check your spam folder and mark the email as not spam!. If you Skip this step, you won't be able to receive order-related updates via email.

How to write an AP Lang synthesis essay: Format + Outline + Tips

synthesis ap lang format

The last component of your basic college English composition course is the AP Lang test. The most crucial—and sometimes challenging—component is the AP Language Synthesis Essay. It's the cornerstone of your future writing abilities, but if you're not prepared, it can be challenging. Additionally, each year's test details could vary. So, with the help of our coursework writers, let's explore what an AP lang synthesis essay is and how to write it. 

What is ap lang?

The AP Language exam is a bit lengthy. Students who want to pass this AP test need to be well-versed in multiple rubrics. Reading and writing are covered in the first half, while three distinct kinds of essays are covered in the second, which is a little more freeform. Of those three, the synthesis essay is the AP Lang rubric, which is the most interesting and, surprisingly, often the most challenging to tackle. We'll concentrate today on the AP Lang synthesis essay, especially to make sure you know what to expect on your test.

Understanding the AP Lang synthesis essay

Do you know what is a synthesis essay ap lang? Imagine someone is selling a mysterious remedy claiming it can cure any disease, but they won't reveal its ingredients or how it works. Would it be wise to buy this remedy? Probably not!

The same procedure applies when writing an essay. If you do not cite your sources for your thoughts, a reader won't believe even your ideas are the best ones. That's where the synthesis essay comes in! In a comprehensive essay, an argument is made on the basis of outside sources. Synthesis essays are used to develop compelling arguments for your viewpoints. You not only need to gather the information, but you also need to add your opinion. 

The goal of the AP synthesis essay rubric is to demonstrate your ability for in-depth source analysis and the creation of an original, well-reasoned thesis statement backed up by relevant evidence. Writing an essay of this kind requires both critical and creative thinking since you have to evaluate the data you have collected and combine it to create a fresh, original viewpoint.

If you find this assignment too complex, you can ask us, ' Write my essay for me ', and our writers will complete it for you.

Types of Synthesis Essays

Synthesis essays are of different types. But the most common are explanatory and argumentative synthesis essays. Let's have a look at them:

Explanatory synthesis essay

The purpose of an explanatory essay is to cover a subject, go into great depth about it, and then show a deep understanding of the subject. You can convince the reader about something without necessarily getting into a furious debate with them. All you have to do is compile an unbiased summary.

Argumentative synthesis essay

An argumentative essay contains personal opinions, but it should be as objective and fact based as possible despite the possibility of bias. You should stick to logical and objective facts when presenting your arguments rather than depending on your feelings. This strategy will strengthen your claim.

Key Components of an AP Lang Synthesis Essay Outline

When you go through the examples of synthesis essays, you will notice that their structure is almost similar to other essays. You have to do an in-depth analysis of the material you gathered before you start working on a synthesis essay ap lang outline. Keep in mind that if the document is properly structured, then it must receive good grades. If you don't format your essay correctly, you won't get good marks even if you write a superb one. You must ensure that you have a thorough understanding of how to write a synthesis essay ap lang.

A lot of time is required to write a synthesis essay outline for the AP Lang test. Simply follow the below steps:

Introduction:

Write about the topic you're going to cover. Add a strong thesis statement. Your thesis statement should cover three ideas on the selected topic. Moreover, it should also include some background details and a strong attention-grabbing hook statement.

The body paragraphs (3 to 4 paragraphs):

As per the standard essay outline, you must write three paragraphs in the body section of your synthesis essay. Start each paragraph with a new thought or idea. If you use someone else's words exactly, put them in quotes. Also, the evidence must be from a reliable source. Any information that is not your own should be cited, and your sources should all be listed in your works cited page or bibliography.

Conclusion (1 paragraph):

This section is dedicated to wrapping up your paper and providing a logical conclusion. Only tie up the topics you covered in the introduction and body, without introducing any new ideas. Your conclusion statement must be strong enough to leave a lasting impression on the reader.

How do you write a synthesis essay introduction?

The most challenging aspect of writing is often getting started. When your mind is teeming with ideas, it can be overwhelming to decide where to begin. Students often spend numerous hours brainstorming ideas. An effective approach is to address this question: 'What would grab my attention if I were the reader"? When you're coming up with ideas for how to compose the paper, keep in mind the specific requirements of the synthesis essay. You can begin in a few different ways. First method is to present a fascinating fact that not everyone is aware of. The second is to start the essay with a well-known quotation related to the subject. The success of your work will mostly depend on the first impression you make on the reader. So, try to grab their interest quickly.

You can also learn “ how to write a research paper introduction ” from our latest guide.

Step-By-Step Guide on How to write a good synthesis essay AP lang

Here are some good techniques for writing a synthesis essays:

Thoroughly read assignment details

Think about the requirements for the assignment. It is very important to understand the assignment completely. Teachers will frequently ask a question regarding a particular subject. They may then assign you a number of readings from articles, academic journals, or other sources to help you gain information. Knowing what the assignment entails will help you make sure you focus on the appropriate material in your readings.

Choose or read about the topic

Read the materials carefully, regardless of whether your instructor assigns you to read a certain amount or requires you to conduct your own research. To fully understand the authors' points of view and their connections, think about annotating each text. One of the most important aspects of synthesis essays is the ability to contrast and compare ideas from multiple sources.

Make a strong thesis statement

Take time to read each source deeply and create a synthesis essay AP lang thesis statement. Do not overlook minute details. The thesis statement is the baseline of your writing. Write it clearly and concisely. Do not overthink too much because you're not adding any details at this point.

Create synthesis essay outline ap lang

Making an outline will assist you in organizing the structure of your essay. You can outline your topic and any supporting details in a structured format with Roman numerals. There may be subcategories inside each where you can put references to the different articles and make particular references to points you will make. By citing these sources, you can make sure that your essay includes passages or details from each article.

Compose synthesis essay introduction

A paper's introduction is its opening paragraph. Its primary goal is to introduce the paper's basic idea, include any background information that is required, and, ideally, grab the reader's interest. You should have a compelling thesis statement in the first paragraph. This is where you will introduce your argument or the point of view you are investigating.

Add body paragraphs to your essay

The three primary components of a strong body paragraph are a topic sentence (also known as a key sentence), relevant supporting phrases, and a closing (also known as a transitional) sentence. This format provides clear, succinct information while maintaining your paragraph's focus on the main point. The various components and justifications of your thesis, together with supporting data from each source, should be covered in detail in each body paragraph. Describe the theme that runs through your materials and how your text relates to them. Add counter arguments as well as how your source material may dispute those claims while supporting your own.

Write conclusion

The last section of a research paper, essay, or article that provides a summary of the entire work is called the conclusion. Your synthesis essay's conclusion paragraph should repeat your thesis, provide a summary of the main points you raised throughout the essay, and provide your assessment of the main idea.

Make a bibliography

An essay's bibliography is a list of the sources you consulted while writing it. You may compose this on a formal work cited or references page at the conclusion of the essay, depending on the essay type. Try including any works that you have quoted or paraphrased in order to provide context.

If you don't know how to cite sources in AP Lang synthesis essay, then avail of our annotated bibliography writing service to cite your sources correctly. 

When writing is complete, go over the assignment once more to see if there are any formatting requirements. Depending on your field, you may frequently submit essays using common formatting styles like MLA or APA. This may change depending on the kind of essay you write because synthesis essays can be written on a wide range of topics.

Recommended Article: “ MLA essay format ” 

Before submitting or presenting your assignment, make sure to proofread it several times. Sometimes, the entire core of an argument or opinion can be altered or discredited by a few misplaced words or grammatical faults. Make sure your syntax, grammar, and writing style are as precise and understandable as you can. This will make you seem like a reliable source.

Synthesis Essay AP lang rubric

Generally, a synthesis essay rubric assesses multiple factors, such as your ability to compile and integrate several sources, your ability to build a solid argument, and your proficiency with language and mechanics. Although rubrics differ from instructor to instructor, a simple AP synthesis essay rubric includes the following:

  • Your argument's cohesion and clarity
  • Ability to back up your claims with proof
  • Language and mechanics used
  • Combining ideas from several sources
  • Information synthesis from several sources

To make sure you're fulfilling all the requirements for the assignment, make sure you go over any rubrics your professor or teacher may have provided. As an alternative, you can hire our team of qualified writers to complete your write my term paper  request without sacrificing any of your own work.

Tips for Synthesis Essay AP Lang

Now that you know how to write a synthesis essay in AP lang properly, your chances of receiving an A are significantly increased. Use these synthesis essay tips in AP Lang to ensure that your project will receive a high score. You can become an expert writer by adhering to them.

Pick the right subject

As a student, you may be motivated to write on a topic about which you are unfamiliar in order to widen your horizons. Learning new things is perfectly acceptable, but it should happen on your own time rather than while you are completing a paper for a grade. Write an essay based on your knowledge; we assure you that you will learn a great deal of new information when you research the topic. Focusing on improving your writing abilities and achieving a good mark should be your main objectives; writing on an entirely different subject makes it more likely that you won't achieve an A.   

Refer credible sources

One of the main issues facing academia today is that new students frequently quote Facebook, blogs, and other unreliable media sources in their papers because they are unsure of what makes an authentic source. When writing a paper, refer only to books, academic journals, the internet, and other similar resources.

Properly cite your sources

Making improper or missing citations and bibliographies will always result in a failing assignment. Failure to adhere to the professor's directions is another factor that frequently gets students into trouble. This frequently occurs when a student chooses to cite in APA or Harvard styles, but the professor prefers another format, like MLA. Never carry out this. Regardless of whether you agree with your professor's instructions or not, you should always obey them.

Read your essay aloud

Speaking out loud while reading an essay can help you identify sentences or paragraphs that need clarification or correction. Try doing this to find any last-minute structural or grammar errors.

Synthesis Essay Do's and Don'ts

  • Leave enough time for proofreading and editing your essay.
  • Make a proper schedule for how much time you are required to write each section.
  • Try to use MLA or Chicago style to format your essay.
  • Add as many arguments as you can to support your claim, but all must be solid and relevant to the topic.
  • Read the source material thoroughly.
  • Private anecdotes should not be used to support your argument unless the essay genre expressly permits it. These don't provide a strong enough case.
  • Don't get too deep into the essay topic; instead, immerse yourself and show your thorough understanding of the subject.

AP Lang Synthesis Essay Prompts

Here is a list of AP synthesis essay prompts:

  • Analyze the impact of technology on modern society.  Use evidence from at least four sources to support your viewpoint.
  • Discuss the government role in climate change.
  • Explore the pros and cons of social media in the context of interpersonal communication.
  • Analyze the relationship between education and income inequality.
  • Consider the effects of globalization on the economy. Synthesize information from at least three sources to make your case.
  • Discuss the effects of climate change on global agriculture. Cite evidence from at least four sources to support your argument.
  • Consider the effects of widespread monitoring programs on people's civil liberties and right to privacy while balancing potential benefits for national security against worries about privacy invasion.
  • Analyze the impact of immigration on cultural diversity in the United States.
  • Discuss the ways that media sources, such as news sites, social media, and advertisements, affect societal attitudes, beliefs, and actions as well as public opinion.

Final Thoughts

Now we've come to the end of the blog; you should not be feeling stressed by the thought of writing an AP Lang synthesis essay. Moreover, you already know that analyzing sources, creating a strong outline, and skillfully integrating them into your writing are essential to success.

So don't be afraid, brave writer! You're ready to start writing a successful paper now because you have these synthesis essay writing strategies at your hand. Alternatively, you may simply buy an essay to wow your teachers!

Table of Contents

Persuasive essay topics – how to choose one for you, how to write a persuasive essay- expert tips.

synthesis ap lang format

Calculate for all schools

Your chance of acceptance, your chancing factors, extracurriculars, help with ap lang synthesis essay.

Struggling with the synthesis essay for AP Lang. Can someone provide me with some strategies or advice on how to best approach this type of essay? Or any sample essays/resources to reference?

I understand that the synthesis essay can be challenging, but with the right strategies and practice, you can definitely improve! Here are some tips to help you approach the synthesis essay in AP Lang:

1. Read the prompt carefully:

Before you dive into the sources, make sure you understand the prompt and know what you are being asked to do. This will help you focus on the main points as you read and analyze the provided sources.

2. Skim the sources:

Take a few minutes to skim through the sources and get a general idea of their content. Don't try to read them in-depth just yet - you'll do that as you begin writing. Keep the prompt in mind as you skim, and focus on identifying the main arguments and perspectives of each source.

3. Develop a thesis:

The synthesis essay requires you to develop a clear and concise thesis, based on the prompt and various perspectives. Your thesis should be an arguable statement that outlines the main points you will discuss in your essay.

4. Analyze and organize the sources:

Devote some time to thoroughly reading and annotating the sources. As you do, determine how each source supports or challenges your thesis. Consider grouping similar sources together to help build your main points and paragraphs.

5. Outline your essay:

Before you begin writing, create an outline to help guide your essay. List the main points and sub-points you'll be discussing, based on your analysis of the sources, and organize them in a logical order.

6. Integrate the sources effectively:

When crafting your essay, make sure to smoothly integrate the sources into your writing. Use direct quotations, paraphrases, and summaries to support your points and ensure you introduce the sources properly (e.g., "According to Source A, ...").

7. Address counterarguments:

To create a well-rounded essay, consider addressing counterarguments or other perspectives that go against your thesis. This demonstrates your ability to think critically and engage with different viewpoints.

8. Revise and proofread:

Once you've written your essay, take the time to revise and proofread it. Make sure your ideas are clearly conveyed, your sources are well-integrated, and your grammar and spelling are perfect.

As for sample essays and resources, the College Board's AP Lang Exam page ( https://apstudents.collegeboard.org/courses/ap-english-language-and-composition) has some sample prompts and student responses, which can be particularly helpful for understanding the expectations of the synthesis essay. Additionally, CollegeVine ( https://blog.collegevine.com/how-to-write-ap-lang-synthesis-essay/) is a great resource for more tips and advice on writing a successful synthesis essay.

Good luck, and I hope these tips help you improve your synthesis essay skills!

About CollegeVine’s Expert FAQ

CollegeVine’s Q&A seeks to offer informed perspectives on commonly asked admissions questions. Every answer is refined and validated by our team of admissions experts to ensure it resonates with trusted knowledge in the field.

IMAGES

  1. How to write a good synthesis essay ap lang

    synthesis ap lang format

  2. AmStud

    synthesis ap lang format

  3. 💄 Synthesis essay template. How to Write the AP Lang Synthesis Essay

    synthesis ap lang format

  4. How To Write The Ap Lang Synthesis Essay

    synthesis ap lang format

  5. Synthesis Essay: Basic Guide on Writing a Good Essay

    synthesis ap lang format

  6. How to Write a Synthesis Essay

    synthesis ap lang format

VIDEO

  1. [LAPIS] ML22120 製品紹介 [音声合成LSI]

  2. Macrolides/Clinical uses & Bacterial targets/By Dr.Hamza Lectures

  3. Toward effective electrocatalytic C–N coupling for the synthesis of organic nitrogeno...

  4. Several temperature-sensitive mutant strains of E: coli display various characteristics Below are o…

  5. League Champions and their cooler versions

  6. Pre-Exam Cram W/ Coach Hall Writes (2023)

COMMENTS

  1. How to Write a Perfect Synthesis Essay for the AP Language Exam

    Step 5: Draft Your Essay Response. The great thing about taking a few minutes to develop an outline is that you can develop it out into your essay draft. After you take about 5 to 10 minutes to outline your synthesis essay, you can use the remaining 30 to 35 minutes to draft your essay and review it.

  2. Synthesis Essay Materials

    The two synthesis essay questions below are examples of the question type that has been one of the three free-response questions on the AP English Language and Composition Exam as of the May 2007 exam. The synthesis question asks students to synthesize information from a variety of sources to inform their own discussion of a topic. Students are given a 15-minute reading period to accommodate ...

  3. How to Write the AP Lang Synthesis Essay + Example

    The AP Lang synthesis essay is the first of three essays included in the Free Response section of the AP Lang exam. The exam presents 6-7 sources that are organized around a specific topic, with two of those sources purely visual, including a single quantitative source (like a graph or pie chart).

  4. How to Write the AP Lang Synthesis Essay with Example

    AP Lang Exam - Scoring. In the second part of the AP Lang exam, students can earn a possible 6 points on each essay. 1 point is earned for the development of a thesis. Up to 4 points can be earned for evidence and commentary. The final 1 point is earned for sophistication of thought. AP Lang Exam - Takeaways

  5. PDF AP English Language and Composition

    AP® English Language and Composition 2022 Scoring Guidelines . Synthesis Essay 6 points . Since the early 2000s, the United States government and a number of corporations have sponsored initiatives to improve education in the STEM disciplines: science, technology, engineering, and mathematics.

  6. Preparing for the Synthesis Question

    There is a 15-minute period allotted to the free-response section to do so. The student will be permitted to read and write on the cover sheet to the synthesis question, which will contain some introductory material, the prompt itself, and a list of the sources. The students will also be permitted to read and annotate the sources themselves.

  7. How to Write an AP Lang Synthesis Essay: Tips & Steps

    In this guide, you will learn the synthesis AP Lang definition, its format, and structure, as well as the essential tips about how to write an AP Lang synthesis essay. ... Overall, a high-scoring AP Language synthesis essay effectively synthesizes multiple sources to develop a nuanced argument that demonstrates critical thinking, coherence, and ...

  8. How to Write the AP Lang Synthesis Essay + Essay Template

    The use of rhetorical devices AP Lang is also pretty important. So once you flesh out your essay a bit, spend some time trying to come up with the perfect wording. Step 5. Finalize. The first finished version of your essay is a draft. Don't be hasty to turn it in. Read over it a couple of times.

  9. Writing a synthesis essay for AP Lang

    Sure, a synthesis essay is a type of essay that requires you to use multiple sources to create an argument. In an AP Lang synthesis essay, you'll typically be provided with the sources and will need to analyze them, identify the main ideas, and then connect those ideas to your central argument or thesis. Here's a step-by-step breakdown of how to write a strong synthesis essay for AP Lang: 1.

  10. AP Lang

    Section II of the AP English Language and Composition exam includes three free-response questions that you must answer in 2 hours and 15 minutes. This guide will focus on Question 1 of Section II of the exam, the. Synthesis question. As with all AP exams with free-response questions, the. Synthesis question.

  11. Acing the AP® English Language and Composition Synthesis Essay

    The newest section of the AP® English Language and Composition Exam, the synthesis essay, is one of three essays you will be completing during the examination's 2-hour free-response period. However, you'll also have a 15-minute reading and planning period just for this essay, and if you use this time to plan effectively, you can't go wrong.

  12. AP English Language Exam Practice: Synthesis Study Plan

    Resources you need to improve your Synthesis essay on the AP English Language and Composition exam. Includes revelant readings and practice problems. Note: For best results, click to highlight and copy/paste this list into your Fiveable Rooms Task Card to automatically create individual tasks. Jumpstart your studying in 5 seconds!

  13. How to Ace the AP Language Synthesis Essay

    In this video, I'll show you how to write the AP English Language synthesis essay (Q1) step by step using the actual 2017 prompt. This tutorial also includes...

  14. Synthesis essay format AP Lang

    Hello! A synthesis essay in AP Language and Composition requires you to analyze multiple sources and develop your own argument based on the information provided. Here's a general outline to help you structure your synthesis essay: 1. Introduction - Start with a hook to grab your reader's attention. - Briefly introduce the topic and provide context for understanding the issue.

  15. AP English Language and Composition Exam

    Section II: Free Response. 3 Questions | 2 hours 15 minutes (includes a 15-minute reading period | 55% of Exam Score. Students write essays that respond to 3 free-response prompts from the following categories: Synthesis Question: After reading 6-7 texts about a topic (including visual and quantitative sources), students will compose an ...

  16. How to write a good AP synthesis essay

    Here's a handy timeline to keep in mind during the 55-minute-long synthesis essay portion of the AP Lang exam: Reading the directions, sources, and prompt: 15 minutes. Analyzing the sources and outlining your response: 10 minutes. Drafting your response: 25 minutes. Reviewing and revising your response: 5 minutes.

  17. Perfecting the Synthesis Essay for AP Lang?

    Hi! It's great that you're working on improving your synthesis essay skills for the AP Lang exam. Here are some tips and best practices to construct an effective argument while incorporating various viewpoints: 1. Understand the prompt: Make sure to read the prompt carefully and understand what is being asked. It might help to rephrase it in your own words to ensure you've fully grasped the ...

  18. AP Lang Exam Guide

    The 2024 AP English Language and Composition exam format will be: Section I: Multiple Choice - 45% of your score. 45 questions in 1 hour. Section II: Free Response Section - 55% of your score. 2 hours and 15 minutes for: 1. synthesis essay. 1. rhetorical analysis essay.

  19. How to Write a Synthesis Essay AP Lang

    Typically, a good AP Lang synthesis essay will have around 3-4 well-constructed and reasoned body paragraphs, but this is just a general guideline. Conclusion Paragraph. Restate your THESIS statement in a new and interesting way (that is, do not simply repeat your thesis word-for-word as it appears in the introduction!).

  20. Need help with AP Lang library synthesis essay

    For an AP Lang synthesis essay, it's important to understand the purpose, which is to integrate multiple sources effectively to support your own point of view. Here are some steps to help you approach your library synthesis essay: 1. Analyze the prompt: Make sure you understand the key question or topic you're exploring in your synthesis essay.

  21. AP English Language and Composition Past Exam Questions

    Download free-response questions from past exams along with scoring guidelines, sample responses from exam takers, and scoring distributions. If you are using assistive technology and need help accessing these PDFs in another format, contact Services for Students with Disabilities at 212-713-8333 or by email at [email protected].

  22. Complete Guide on AP Lang Synthesis Essay

    Synthesis Essay AP lang rubric. Generally, a synthesis essay rubric assesses multiple factors, such as your ability to compile and integrate several sources, your ability to build a solid argument, and your proficiency with language and mechanics. Although rubrics differ from instructor to instructor, a simple AP synthesis essay rubric includes ...

  23. Help with AP Lang Synthesis Essay?

    Here are some tips to help you approach the synthesis essay in AP Lang: 1. Read the prompt carefully: Before you dive into the sources, make sure you understand the prompt and know what you are being asked to do. This will help you focus on the main points as you read and analyze the provided sources. 2. Skim the sources: